You are on page 1of 92

Table of Contents

Table of Contents 1
Episode 97: “Most Important ” 2
Episode 239: OBGYN Risk Factors 3
Episode 250: HY Vaccine 4
Episode 251: The HY Thyroid Podcast 8
Episode 255 Legendary Step 2CK/3 Pharmacology 13
Episode 286 ULTRA HY ARDS Podcast 17
Episode 287 ULTRA HY Urinalysis Podcast 19
Episode 289 Ultra HY Alcoholic Podcast 21
Episode 304 Floridly HY Ortho/Trauma Podcast Part 2 27
Episode 305 Upper Limbs Rapid Review 1 30
Episode 308 Floridly High Yield NBME Cortisol 33
Episode 310 Floridly HY Knee Exam and Pathologies 38
Episode 313 Floridly HY Hyperkalemia 43
Episode 325 Extremely HY Screening Guidelines 46
Episode 339 HY Pulmonary Embolism 52
Episode 379 A Series of HY “Elderly Vignettes” 55
Episode 381 Some HY Pharmacology Vignettes 59
Episode 404 Floridly HY CXR (Chest X-Ray) 61
Episode 424 HY Rules For Remembering Inheritance Patterns 74
Episode 453 Quick Review HY Psych Timelines 75
Episode 455 HY Autosomal Dominant Disorders 77
Episode 460 Floridly HY Antibiotic Review I 77
Episode 461 Floridly HY Antibiotic Review II 80
Episode 97: “Most Important ”

MI = most important
MIPF = most important prognostic factor
#1 RF = most important risk factor

1. MIPF in retinal detachment → time to surgical intervention to fix


2. MIPF of Breast Ca. → 1. number of axillary LN “spread”. 2. Size of tumor
3. MI predictor of survival in aortic coarctation → Age of defect repair
4. #1 RF that predisposes to rupture of AAA → diameter of aneurysm
5. #1 RF for development of SCC of skin → cumulative exposure to sun
6. MIPF of melanoma → depth of invasion. “Breslow depth”
7. MIPF in mesenteric ischemia → bowel infarction
8. MI measure in prevention of HCC → Hep B vaccine
9. #1 RF of development of ovarian Ca → FHx of Ovarian Ca.
10. MIPF of cervical cancer → 1. stage at which it’s diagnosed. 2. Involvement of
pelvic/paraaortic LN
11. #1 RF of prostate ca → Age
12. #1 RF of bladder ca → smoking
13. MIPF in determining outcome of treatment in colon ca → Stage at diagnosis
14. MI test needed in dx of IE → Blood culture
15. MI complication of peritoneal dialysis → peritonitis
16. MI goal in mgmt. of COPD → smoking cessation
17. #1 RF for OSA development → obesity
18. MI disease associated with central sleep apnea → heart failure
19. MI non-pharmacologic intervention in mgmt. of osteoarthritis → exercise
20. MI modifiable RF in development of osteoarthritis → obesity
21. 2 #1 RF for development of C. diff colitis → hospitalization and Abx
22. MI opportunistic pathogen in transplant pt → CMV
23. #1 RF for development of cholangiocarcinoma → primary sclerosing cholangitis
24. 2 #1 RF in development of HCC → Cirrhosis + Hep B infx
25. #1 RF in dev of lung disease related to asbestos → cumulative exposure to asbestos fibers.
26. MI therapeutic step in mgmt. of acute pancreatitis → IV fluid resuscitation
27. MI physical exam finding in dx of AS → pulsus parvus et tardus (delayed upstroke in carotid
pulse)
28. MIPF in pt with CAD → Left ventricular function
29. Most important factor in preventing lung injury when using ventilator → mechanical
ventilation with lung protective strategies. I.E low TV.
30. MI oncologic association in long term celiac disease → small bowel lymphoma (enteric
associated T-cell lymphoma)
31. MC disease of the retina in elderly → macular degeneration
32. #1 RF in development of pressure ulcers → immobility
33. MI physical exam finding in dx of RAS → presence of abdominal bruit
34. MIPF in preserving neurologic function in spinal cord compression → early diagnosis
35. MI factor in determining TB treatment success → adhering to treatment regimen.

Association of infectious agents with neoplasms

1. Cervical/anal/head and neck/vocal cord cancers → HPV


2. Kaposi Sarcoma → HHV-8
3. Bacillary angiomatosis → Bartonella henselae
4. Hodgkin’s Lymphoma → EBV
5. Burkitt’s Lymphoma → EBV
6. Lymphomas after transplant → EBV
7. MALToma → H pylori
8. HCC → Hep B + C

CROSS CHECKED? YES

-------------------------------------------------------------------------------------------------------------------------------
Episode 239: OBGYN Risk Factors

1. MI risk factor for postpartum depression→ hx of depression


2. Most reliable indicator of successful labor induction→ bishop score
3. MCC of infertility→ ovulatory dysfunction (PCOS)
4. MC late adverse effect of pelvic radiotherapy→ vaginal stenosis and it is the biggest
risk factor for uterine sarcoma.
5. MC risk factor for squamous cell carcinoma of vaginal→ HPV
6. Most important RF for clear cell carcinoma → maternal exposure in utero to DES. As
well as t shaped uterus (2ndtrimester losses)
7. Most important RF for vulvar carcinoma→ HPV if not in the options lichen sclerosus.
Do punch biopsy
8. Biggest RF for postpartum endometritis → having a C-section
9. Biggest RF for pelvic septic thrombophlebitis→ having a history postpartum
endometritis
10. MI of prognosis of breast cancer→ involvement of axillary lymph nodes
11. Predictor of worse prognosis in gestational trophoblastic disease→ mets to liver and
brain, higher ß hcg, more time between pregnancy and gestational trophoblastic disease
12. MI predictor of prognosis of vulvar cancer→ lymph nodes then lesion size
13. MI prognostic factor in endometrial cancer is stage
14. MC presenting complaint is vaginal bleeding
15. Biggest RF for endometriosis → family history of endometriosis
16. MC cause of DIC in pregnancy→ abruptio placentae
17. Biggest Rf for placenta previa→ prior C section
18. Biggest RF of placental abruption→ prior history of placental abruption or
hypertension
19. Biggest RF for preterm labor→ previous history of preterm labor, ureaplasma or
gardnerella vaginalis
20. Biggest NBME risk factor for cervical incompetence→ LEEP or conization
21. Most important intervention to prevent NEC, intraventricular hemorrhage or RDS→
betamethasone administration.
22. Preventive measure in a woman in PPROM to prevent infection→ decreasing
cervical exam numbers
23. MI Rf of IUGR→ previous history of same
24. Macrosomia Rf→ maternal diabetes
25. Biggest NBME Rf for fetal tachycardia→ maternal fever
26. Biggest RF for postpartum hemorrhage→ atony
27. PDA→ premature
28. Biggest Rf of ectopic pregnancy→ previous history of same
29. Uterine rupture biggest Rf→ scar from prior c section
30. Breast cancer RF→ age
31. Biggest RF of preeclampsia → prior history of same

32. Biggest RF for uterine inversion→ prior history of same


33. Biggest risk factor for chorioamnionitis → prolonged rupture of membranes (>18 hrs
give GBS ppx).
34. 2 biggest RF for gestational trophoblastic disease→ nulliparity and extremes of age
35. GBS PPx with intrapartum penicillin→ child with GBS sepsis regardless of culture,
positive urine culture at any point in pregnancy, if unknown status, if the woman had
ruptured membranes for ≥ 18, or intrapartum fever.
36. Rhogam→ at 28 wks, at any procedure, within 3 days postpartum, if you do cerclage,
ectopic pregnancy, MVA. Or any other thing that can cause mixing.

CROSS CHECKED? YES


------------------------------------------------------------------------------------------------------------------------

Episode 250: HY Vaccine


● Two big types of vaccines
○ Live attenuated
■ Bug that has been severely weakened, but retains ability to infect
cells.
■ Generates a T-Cell response
■ No real need to give boosters
■ Ex
● MMR
● Varicella
● Intranasal influenza vaccine
○ Killed/Inactivated vaccine
■ Destroyed bug but you keep antigenic parts, which generates a
response from immune system
■ Generates Humoral response
■ Need to give boosters
○ Toxoid vaccine
■ Toxin from bug that can act on receptors in the body to generate
antibody response
● Pt is a newborn with a mom that has HepB surface antigens +
○ NBS
■ HepB vaccine AND HepB immunoglobulin
● Administered In opposite extremities
● Rotavirus vaccine is live attenuated (with a twist)
○ Live attenuated are not usually given to those < 1y/o
■ Rule does NOT apply to newborns.
■ Kids receive a vaccine at 2,4,6 months.
● Child that receives rotavirus vaccine. Which of the following is the most
dangerous complication?
○ Intussusception
■ If pt has had it in the past intuss
■ Hx of meckel’s
■ IgA nephropathy (or HSP)
● What do these vaccines have in common: TDAP, HepB or PCV-13, polio, HepA,
neisseria meningitidis?
○ Inactivated
● Two types of influenza
○ Inactivated
■ Intramuscular
○ Live-attenuated
■ Intranasal. Inhaled as mist
● PCV-13 and PPSV-23
○ PCV-13
■ Conjugated. Pneumococcal “C”onjugated
● B + T cell response
○ PPSV 23
■ Polysaccharide
● Humoral response (No T cells)
○ PCV 13 always taken FIRST before 23
■ Mnemonic aid: 13 comes before 23.
○ PCV13 is taken by everybody!
○ PPSV 23
■ Given for those > 65 y/o
■ Those under 65 y/o that should have vaccine include (think of
organs with chronic disease):
● Smoker
● Diabetes
● Cochlear implant
○ Older person on test.
○ Alport syndrome
● Chronic CSF Leak
● Immunodeficiency (HIV, Bruton, SCID, CVID)
● Those with no spleen
● Who does not get live attenuated vaccines?
○ Pregnant women
○ < 1 y/o (exception: rotavirus)
○ CD <200 (immunodeficiency)
● Vaccines in kids
○ Approximate rule but works a lot:
■ Most pediatric patients are at least 3 doses
● But not always true
■ Most are given at 2, 4, 6 months of age.
● Those that don’t follow this rule?
○ Live attenuated vaccines!
○ Hep B vaccine
■ Need to get it before you leave the hospital.
■ Three doses
● First dose at birth
● Second dose: within 2 months of birth
● Third dose is between 6-18 months of age
○ Rotavirus vaccine obeys second rule
■ 3 doses vaccines given at 2,4, and 6 months.
○ DTap vaccine
■ 5 doses with extra information to know.
● 3 doses at 2, 4, and 6 months
● 4rth dose: ~ 15-18 months of age
● 5th dose: 4-6 y/o
■ When you’re older:
● TDap (Adolescent
● TD booster
○ H. Flu Type B vaccine
■ 3 doses
● 1st dose: 2 months
● 2nd dose: 4 months
● 3rd dose: 12-15 months
○ PCV-13 vaccine
■ 4 doses
● 1st dose: 2 months
● 2nd dose: 4 months
● 3rd dose: 6 months
■ 4th dose: 12 - 15 months
● Polio vaccine
○ Four doses
■ 1st dose: 2 months
■ 2nd dose: 4 months
■ 3rd dose: 6 months
■ 4rth dose: 4-6 y/o
● Influenza vaccine
○ Every year.
○ Less than a year old do NOT give intranasal.
● Specialized live-attenuated
○ MMR and varicella
○ Group both together with the same vaccine schedule.
○ Two doses
■ 1st dose: 12-15 months
■ 2nd dose: 4-6 y/o
● Hep A vaccine
○ Two doses
■ 1st dose within 2 y/o
■ 2nd dose: 6 months after 1st dose
○ Later given in adolescence
● 10-18 y/o
○ Meningococcal vaccine
■ Two doses
● First dose: 11-12 y/o
● Second dose: 16 y/o
● HPV vaccine
○ Can give starting from 9-26 y/o
● Zoster vaccine
○ Not the same as varicella
○ Get zoster at age of 50
■ Two doses (6 months apart)
● Pregnancy
○ Women supposed to get TDAP vaccine
■ Within 27-36 weeks
● Pt splenectomy (Sickle cell, trauma)
○ PCV-13 and PPSV-23, strep pneumo, H flu, and
neisseria meningitidis
● Timeline between PCV-13 and PPSV-23
○ Wait between two months between both vaccines
● Pt. has sickle cell/splenectomy in addition to above those
patients need
○ Amoxicillin or penicillin until they reach the age of 5.
● Egg allergy
○ Avoid Yellow fever (Everything else is fine, yes - even
MMR!!)
----------------------------------------------------------------------------------------------------------------------------

Episode 251: The HY Thyroid Podcast


● Embryo
○ Thyroid comes from base of tongue
■ Foramen cecum (Endoderm)
● Thyroglossal duct props down from that
● Pt with a neck mass at midline that moves with swallowing?
○ Dx?
■ Thyroglossal duct cyst
■ If nbme is like “they have a neck mass and it is lateral to midline, does not
move with swallowing” think branchial cleft cyst, not thyroglossal duct
cyst!
● Branchial cleft cysts are lateral, don't move with swallowing,
derived from pharyngeal ectodermal grooves/clefts
● Thyroglossal duct cysts are midline, move with swallowing,
derived from endoderm
● Pt notices a lump on the neck when putting on tie, next best step?
○ Nbme likely describing thyroid nodule, best step is get TSH Level
■ If TSH is low, there must be some hyperthyroid going on, so it's a “hot”
nodule
● If “hot” nodule, get RAIU scan
○ If global uptake in ENTIRE thyroid it’s Graves disease
■ Graves is the number one cause of hyperthyroid
disease
■ To treat give Methimazole/PTU or radioactive
thyroid
■ Don't pick thyroidectomy that's usually wrong
answer
○ If RAIU shows one hot spot, thats toxic adenoma
■ Why is there only one hotspot and why isn't the
whole thyroid lit up?
■ Well cause there's one small group of cells
secreting T3/T4, which goes on to suppress TSH,
hence other areas of thyroid aren't stimulated
○ If multiple hot spots? Toxic multinodular goiter
■ If TSH is anything besides low (ie normal or high) it is a “cold” nodule
● If cold nodule next best step is US then FNA
● 75% most people with cold nodules just have colloid cysts which
are nothing to worry about, but 20% could have thyroid cancer,
only way to tell is biopsy

● PT gets CT for some reason and CT says they have no thyroid but they feel fine and
TSH and thyroid levels normal, what's going on?
○ Lingual thyroid
■ Remember migration pattern of embryology for this
■ Most common location of ectopic thyroid is the tongue

● You see a newborn with hypothyroidism, what is the most common cause of this?
○ Thyroid dysgenesis (ie where thyroid doesn't properly form)
○ Baby will have large tongue, umbilical hernia
○ Side note: PTU/Methimazole in mom can suppress babies thyroid (they are
teratogenic afterall and cause aplasia cutis), don't freak out when you see these
babies hypothyorid levels, just chill for a few days and repeat exams and labs
later, they'll be fine once those drugs are out of their system

● PT is hypertyroid but RAIU shows no uptake at all, they recently had an URI and their
thyroid is tender, diagnosis?
○ Subacute Thyroiditis (also called De Quervains thyroiditis or granulomatous
thyroiditis, so it has multiple names to confuse you lol)
○ How to differentiate between De Quervains and a person who is taking
exogenous thyroid factiously (i.e. like a person trying to lose weight)?
■ So in DeQuervains there is a dump of thyroid hormone leading to
suppressed TSH and hence cold RAIU
■ Likewise exogenous thyroid also suppresses TSH and leads to cold
thyroid
■ The nbme won't be nice and say “hey they just had a URI and now their
thyroid is tender” so how to tell the difference?
■ Use Thyroglobulin (kind of like using “c-peptide” in insulinoma),
exogenous thyroid will not have thyroglobulin but pts with De Quervains
will have it (ie increased levels of it i think)

● PT is hyperthyroid, RAIU is cold, they have adnexal mass also, diagnoses?


○ Struma Ovarii (makes thyroid hormone)

● PT has neck mass, biopsy reveals xys, what is the biggest Risk Factor for this condition?
○ Prior history of head or neck radiation!
■ Like they had hodgkins as a kid and got radiation, now have neck mass
■ Especially Papillary, which is the most common type of thyroid cancer,
● Papillary = Psammoma bodies, orphan annie nuclei on histology

● PT has Thyroid Cancer that metastasis to the brain, what type of thyroid cancer do they
most likely have?
○ Follicular Thyroid Cancer as it spreads hematogenously!
■ Papillary spreads through lymphatic channels so less likely to go to brain
then Follicular
■ (example like choriocarcinoma spreads hematogenously and goes
everywhere quick)
■ Papillary thyroid cancer has great prognosis unlike follicular

● Is FNA enough to diagnose Follicular Thyroid Cancer?


○ Nope
○ FNA can't tell the difference between Follicular Adenoma and Follicular
Carcinoma, so what is the next best step in diagnosis ?
■ Lobectomy, take out entire thyroid lobe to make sure
● Pt has neck mass, they are having seizures, calcium is like 6, ECG shows QT
prolongation, they have had several family members die from neck masses, what's your
diagnosis?
○ MEN 2A or 2B leading to Medullary thyroid cancer
○ Medullary thyroid cancers tumor marker is Calcitonin (tones down Ca) hence
symptoms
● Patient has MEN 2A or 2B what surgical procedure is indicated in these patients
○ prophylactic thyroidectomy, if you have MEN2A or 2B its not a matter if you'll get
medullary thyroid cancer it is a matter of when
● Pt has thyroid cancer, it lights up on congo red staining, diagnosed?
○ Medullary Thyroid Cancer (derived from cancer of C cells)

● Pt is like 70 (ie they're old, not like 30-40s) has widespread, firm neck, diagnoses?
○ Anaplastic thyroid cancer
○ Awful prognosis :(

● Pt is a 25 yo they're super sick, they've been in ICU, or they had bad sepsis, they're TSH
is fine, T4 is fine, T3 is low, diagnoses?
○ Euthyroid Sick Syndrome
○ Whenever someone that is very sick think this
○ They'll have normal TSH, normal T4, decreased T3, and increased reverse T3
(HY)
○ If patient is sick, losing a lot of weight, body needs to conserve energy, one way it
does this is by decreasing metabolic rates, via decreasing thyroid level (T3)

● Most common cause of hypothyroid on any human (not newborn) you see on an NBME
exam?
○ Hashimotos
■ T3 and T4 low, TSH is high
■ Usually question stem will give you history of some other autoimmune
disease
■ What if PT with hx of Hasimotos has rapidly expanding neck mass, biopsy
shows germinal follicles? Diagnoses?
● Thyroid Lymphoma

● Pt has a history of Graves, they are decompensating, super hypertensive, super


tachycardic, AMS, diagnoses?
○ Thyroid Storm
○ What's the first thing to give these PTs?
■ Beta Blocker (propranolol)
○ What's the second step?
■ Give PTU
○ Then what can you do?
■ Can give SSKI, which is super saturated potassium iodide, which takes
advantage of the Wolff-Chaikoff effect which is the concept that giving a
punch of iodine will transiently shut down thyroid hormone synthesis
○ Can also give steroids

● Signs of Hypothyroid
○ Low heart rate (seen in a lot of nbme exams)
○ Decreased fertility, because if hypothyroid, TRH will go up, which increases
Prolactin, which decreases GnRH (sidenote antipsychotics ie dopamine blockers
work through similar mechanism to decrease fertility)


----------------------------------------------------------------------------------------------------------------------------
Episode 255 Legendary Step 2CK/3 Pharmacology
● Blood drugs
○ antiplatelet/ anticoagulants
○ Anti-platelet
■ Who?
● Stroke or carotid stenosis (TIA)
○ 1st line drug?
■ Aspirin
○ 2nd drug
■ Clopidogrel
○ 3rd drug
■ Dipyridamole
● Peripheral Arterial disease
○ Aspirin for life
● Percutaneous coronary intervention (MI)
○ Dual antiplatelet agents for a few months after the stent is
placed
● Acute MI / Chest pain
○ 1st line drug?
■ Aspirin
● Kawasaki
○ Aspirin + IVIG
● Preeclampsia in previous pregnancy and prevent in future
pregnancy
○ Aspirin prophylaxis
● Woman with cervical insufficiency. How to prevent insufficiency in future pregnancy?
○ Progestin suppository
● Primary Hemostasis
○ Adhesion step
■ GP1B - VWF binding
● Begin to eject ADP from platelet
○ ADP binds to ADP receptor (P2Y12)
■ GPCR triggers signaling cascade
○ Aggregation step
■ GpIIb/IIIa binds to other GpIIb/IIIa
○ Formation of platelet plug
● What does aspirin do?
○ Irreversible COX-1 and COX-2 inhibitor
■ Can’t make TXA2 (helps with platelet aggregation) so you can’t aggregate
● GpIIb/IIIa inhibitors
○ Abciximab, eptifinabie, terafibine
■ Prevent GpIIb/IIIa from binding to another GpIIb/IIIa can’t form platelet
plug
● ADP receptor antagonists (P2Y12 receptor blockers)
○ clopidogrel ,
■ Block receptor ADP won’t bind to it and can’t activate the receptor.
● PD3 inhibitors
○ Cilostazol, dipyridamole
○ Increase cAMP
■ Increase cAMP in platelet INHIBIT signaling cascade downstream of p2y2
receptor
■ P2Y2 is inhibitory GPCR. less camp = more platelet activation. So if you
inhibit PD3 leads to more cAMP and decreased platelet activation.
○ When cAMP is high it causes smooth muscle?
■ To Relax!
● Vasodilation
○ Cilostazol and NBME?
■ Give after trying a supervised exercise program in patients with Peripheral
Arterial Disease.
○ Dipyridamole
■ Pharmacological stress test
■ Boosts levels of adenosine
● Adenosine deaminase inhibitor
○ Causes vasodilation
● Theophylline antagonizes Adenosine

■ ~12 mins
○ Warfarin
■ Prosthetic Heart Valve indication
● Target INR 2.5 - 3.5
● INR 6 and 7
○ Stop the Warfarin, give oral vitamin K
○ Bleeding Out? Reverse with 4FPCC
○ Reverse Heparin with Protamine Sulfate
■ A. Fib from valvular cause i.e. MS cause
■ Risks
● Don’t give to pregnant woman
● Duodenal hematoma (SBO), rectus sheath hematoma, intracranial
bleed
● Don't’ give tPA with warfarin (they’ll bleed out and die)

■ Inhibits Vitamin K Epoxide Reductase


● Can’t gamma carboxylate 2,7,9, 10, Protein C, S
● Protein C and S are anticoagulation
● 2,7,9,10 are coagulants
■ Bridge with Heparin First!
● Activates antithrombin III
○ Can inhibit factors 10 and 2
● Protein C and S have a shorter half life
○ Fall out of circulation first
○ 2,7,9,10 are left for temporary coagulation
○ So there is a temporary hypercoagulable state in patients
with Warfarin, that’s why they need that heparin bridge
● Low Molecular Weight Heparin
○ Also activates antithrombin III BUT only inhibits Factor 10
○ Factor 10 Inhibitors
■ All have Xa in their name
○ Factor 2 inhibitors
■ Argatroban, Dabigatron, Bivalirudin
■ Pt. presentation
● Platelets drop post giving heparin
○ Stop Heparin, don’t give LMWH
○ Start them on Factor 2 inhibitors
○ Dabigatran is reversible by Idarucizumab
● Cardio Drugs
○ ACE inhibitors (-pril)
■ Site of action → pulmonary capillaries, endothelial cells
■ Used in hypertensive diabetic patients
■ Improves survival in HF patients (also beta-blockers and potassium
sparing diuretics)
■ Contraindicated in patients with bilateral renal artery stenosis (Significant
rise in creatinine after ACE inhibitor → renal angiography)
■ Hyperkalemia counteracts the Hypokalemia caused by loop diuretics
■ Scleroderma renal crisis (steroids are never the right answer for
scleroderma)
■ Teratogenic → renal dysgenesis, oligohydramnios
○ Cardiac resynchronization therapy
■ EF < 35% + maximal medical therapy
■ Can improve survival
○ Spironolactone
■ Treat Hirsutism associated with PCOS
■ Used to decrease portal HTN in ascites along with Furosemide for chronic
use
■ Conn syndrome, bilateral adrenal hyperplasia
■ Improves survival in HF
○ Beta-blockers
■ HCOM, improved survival in HF, migraines, akathisia
■ Initial DOC for thyroid storm is Propanol (peripheral conversion of
Thyroid)
■ Labetalol (mixed alpha and beta blocker)
■ Hypertensive moms love Nifedipine
● H = Hydraline, M= alpha methyldopa, L = Labetalol, N = Nifedipine
● Pregnant anti-HTN drugs
■ Don’t give to patients with acute cocaine intoxication
■ Can be used to treat glaucoma (Timolol, etc)
■ Overdose → Atropine then Glucagon
■ Class II antiarrhythmic
○ Alpha-1 blockers
■ Prazosin - PTSD for nightmares (first-line is SSRI and psychotherapy)
■ Useful for BPH
■ Side effect → orthostatic hypotension
■ Tamsulosin (Blocks specifically alpha-1 receptors in bladder)
○ Alpha-2 blockers: Mirtazapine -> used for depression
○ Alpha-2 agonists : Clonidine (leads to rebound HTN)
■ Third-line for ADHD
■ Tourette’s
■ Can be used in opioid withdrawal
○ Loop diuretics
■ Peripheral edema, nephrotic syndrome, CHF
■ Loops lose Calcium → hypocalcemia, hypercalciuria (risk factor for
nephrolithiasis)
■ NKCl(x2) transporter in thick ascending loop of Henle
■ Ototoxic (other ones are Cisplatin, Vanc, Aminoglycosides - Gentamicin)
■ Taking these is like having Bartter Syndrome
○ Thiazide
■ Hypercalcemia with HYPOcalciuria
■ Stronger association with hyponatremia
■ Metolazone is an example
■ Taking these is like having Gitelman Syndrome
○ Amiodarone
■ V Tach
■ Class III - K channel blocker
○ Antiarrhythmics
■ Procainamide - Class Ia, used for WPW, can cause drug-induced lupus
○ Verapamil → can cause hyperprolactinemia
○ Nimodipine → post-stroke vasospasm
○ Digoxin
■ Na-K ATPase blocker
■ Normal job - 3 NA leaves cell, 2 K in → hyperkalemia side effect
■ If patient is hypokalemic, more susceptible to Digoxin toxicity
■ Abdominal pain, yellow vision, diarrhea, etc
■ Contraindicated in WPW
○ Causes of dilated cardiomyopathy
■ Trastuzumab
■ Doxorubicin
■ Clozapine → Myocarditis

----------------------------------------------------------------------------------------------------------------------------

Episode 286 ULTRA HY ARDS Podcast


ARDS (Acute Respiratory Distress Syndrome): Respiratory failure that happens acutely, Many
different causes of ARDS
Most Common Causes: Sepsis, Pancreatitis, COVID-19, Aspiration (stomach contents),
Motor Vehicle Accident (pulmonary contusion), Drowning
Presentation: Tachypnea (rapid breathing), super cyanotic
Dx: Chest X-ray --> Bilateral opacities on imaging (white out on BOTH sides of the lungs)
40% of patients with ARDS will die (best medical efforts, mechanical ventilation, etc..)
More severe --> more likely to die

Pathophysiology of ARDS
1st event (Injury in lungs) --> inflammation (due to recruitment of neutrophils, T-cells) -->
inflammatory mediators released (histamine, bradykinin) --> decrease capillary permeability,
increase vascular permeability --> fluid leaks into lungs (alveoli) --> begin to form hyaline
membrane (diffuse alveolar damage) --> increase distance/thickness O2 must travel to perform
gas-exchange (diffusion ability decreases) --> Oxygen tension inside alveoli lower than in
pulmonary capillaries

Mechanism of Hypoxemia in ARDS:


Increased thickness of alveolar membrane and increased diffusion distance
Intrapulmonary Shunt (no gas exchange occurring due to hyaline membranes and
edema) --> blood flows through artery to vein and is NOT oxygenated

Pulmonary Values:
DLCO (in ARDS): DECREASED
A-a Gradient (in ARDS): INCREASED

Ptx presents with 3-month history of Shortness of Breath, chronic epistaxis, serpiginous tracts
on buccal mucosa: Hereditary Hemorrhagic Telangiectasia (Osler-Weber-Rendau Syndrome)
Pathophysiology: Excessive number of AV malformations (AVMs) --> pulmonary AVMs
leads to DECREASED gas exchange --> hypoxemia (intrapulmonary shunts) --> High Output Heart
Failure

High Output Heart Failure: Type of Dilated Heart Failure (due to overwork of heart muscle)

Diagnosis of ARDS: BERLIN CRITERIA


Acute onset of symptoms
Bilateral opacities on imaging (chest x-ray/chest CT)
Pulmonary Edema (confirm it is NOT due to heart failure; measure PCWP < 18)
PaO2: FiO2 ratio 350-450 (normal); Mild: 201-249; Moderate: 101-200; Severe < 100
PEEP for ARDS patient: < 5

PCWP > 18: Cardiogenic Cause of Pulmonary Edema

Treatment: Aggressive (ICU)


Endotracheal Intubation & Mechanical Ventilator
Mechanical ventilation: PEEP > 5 cm H2O (Minimum), monitor Mean Airway Pressure,
Plateau Pressure (pressure applied during inspiration) < 30 cm H2O, Low Tidal Volume (6
ml H2O/kg using ideal body weight), PCO2 increases

3 types of alveoli:
perfectly normal----middle ground; partially filled with fluid ----destroyed/damaged alveoli
PEEP helps recruit “middle ground” alveoli to work better

3 measures to increase survival in ARDS: Low Tidal Volumes on mechanical ventilators, ECMO
(extra-corporal membrane oxygenation), Placing people in Prone position (laying on belly)

IF plateau Pressure HIGH --> LUNGS CAN EXPLODE

Ptx with carbon monoxide poisoning, carboxyhemoglobin levels elevated initially


Tx: Hyperbaric Chamber
Ptx removed from hyperbaric chamber --> hypoxia, shortness of breath, CXR: Pneumothorax
(black on one side where lung is supposed to be)
Too much oxygen and high pressures --> Pop Lung

Nitric oxide doesn’t do anything (trick on tests)


-----------------------------------------------------------------------------------------------------------------------
Episode 287 ULTRA HY Urinalysis Podcast
Person with SIADH (syndrome of inappropriate Anti-diuretic Hormone)
Causes: Paraneoplastic Syndrome, SSRI user, Pneumonia, Brain process (cryptococcal
meningitis)
Urinalysis: Urine Osmolality HIGH (relative to serum osmolality); Specific Gravity > 1.012

Specific Gravity 1.012 (normal) (less than 1.012 = dilute; greater than 1.012 = concentrated)

Causes of Increased Specific Gravity:


SIADH, Volume depleted (prerenal azotemia), Dehydration, Urine SUPER hypertonic (i.e Diabetic
crisis, DKA/HHS; Contrast for imaging procedure),
Cyclosporine use (hypoperfusion of afferent arterioles), chronic NSAID use (excess
prostaglandins leads to hypoperfusion of afferent arterioles)

Causes of Decreased Specific Gravity:


Excess hydration (psychogenic polydipsia), Diabetes Insipidus (Central=low ADH
production/Peripheral=no response to ADH secretion), Sickle Cell Disease (nephropathy -->
cannot concentrate urine), Intrarenal/Postrenal AKI (dead tubular cells --> cannot concentrate
urine)

Cannot take metformin if going to get procedure requiring contrast dye


Destroy kidneys from contrast & taking metformin can cause life-threatening acidosis

RAAS activation due to hypoperfusion of afferent arterioles --> JGA release renin --> activate
RAAS --> ADH secretion & Mineralocorticoids (i.e. aldosterone) from zona glomerulosa

CHF: Effective Arteriole blood flow REDUCED --> hypoperfusion of Kidneys --> activation of RAAS
--> net constriction of vessels in body --> INCREASE SVR

How to determine WHICH RTA (renal tubular acidosis)


Is potassium high? YES --> Type IV RTA (low aldosterone state --> hyperkalemia)
Is potassium low? Yes; Look at Urine pH
Urine pH > 5.5 --> Distal RTA (Type I RTA)
Urine pH < 5.5 --> Proximal RTA (Type II RTA)

Causes of Type IV RTA (normal anion gap metabolic acidosis):


Spironolactone/Eplerenone use, Addison’s Disease (destroyed adrenal cortex),
Waterhouse-Friedrichsen Syndrome (Due to N. Meningitidis), CAH (21-Hydroxylase deficiency)

Which CAH associated with LOW levels of aldosterone with NO Type IV RTA: 11-β Hydroxylase
deficiency

RTA Type I: Alpha-intercalated cells do not function properly

Vaginal pH:
Trichomonas pH > 4.5
Candida pH < 4.5
Gardnerella Vaginalis pH > 4.5

Low urine pH (overabundance of protons): Increased risk of “acidic” kidney stones


Cysteine Stones; Uric Acid stones

High urine pH (≅ 7): Struvite stones (ammonium-Magnesium-Phosphate) Stones, Calcium


Oxalate stones

Basic Urine: Nephrolithiasis

Basic urine: Urease + bugs: Proteus Mirabilis (staghorn calculi; swarming motility); Staph
Saprophyticus (#2 cause of UTIs, E. Coli #1), Ureaplasma Uryliticum (high urine pH)

Looking for Blood in Ptx urine:


If 3+/4+ blood on urinalysis --> 30-50 RBC per HPF (hemoglobinuria)
Nephrolithiasis, Kidney Injury, Nephritic Syndrome (dysmorphic Erythrocytes), Renal Cell
Carcinoma/Bladder Cancer (RBCs in urine), Wilm’s Tumor (hematuria), IgA nephropathy
(child with URI 2-6 days ago), PSGN (child with URI 2-4 weeks ago)

If 3+/4+ blood on urinalysis --> 0-5 RBC per HPF (Rhabdomyolysis)


Processes that cause muscle breakdown

Glucose in Urine: Plasma glucose high (>180 mg/dL)


Fanconi Syndrome (type II RTA, glucosuria), Diabetes Mellitus
Glucosuria a RF for:
Candida infection, Necrotizing fasciitis of perineum (Foney’s Gangrene), SGLT-2 inhibitor
Nitrite Negative urine + Symptoms of a UTI --> Treat for UTI (nitrites NOT specific)

Squamous cells in patient’s urine --> Contaminated specimen

Hyaline casts in patient’s urine --> Urine concentrated

Pigmented “muddy-brown” granular casts --> ATN/Intrarenal AKI

RBC casts --> Nephritic syndrome

WBC casts --> Pyelonephritis

Eosinophils in urine --> Acute Interstitial Nephritis

Urine Crystal shapes


Calcium oxalate stones (Crohn’s disease --> increased reabsorption of oxalate; ethylene
glycol poisoning) --> ENVELOPE SHAPED/DUMBBELL SHAPED stones
Calcium Phosphate stones (Type I RTA) --> Star shaped/Needle Shaped/Prism shaped
Uric Acid Stones (Tumor Lysis syndrome (treated with chemotherapy for cancer --> cells
die --> uric acid overproduction)) --> Rhomboid Shape
Struvite Stones (Urease + bugs; PUS (Proteus mirabilis, Ureaplasma Uryliticum, Staph
Saprophyticus) --> coffin-lid shaped
Cysteine Stones (COLA transporter defect) --> Hexagonal shaped

Tx of Tumor Lysis Syndrome: Rasburicase, Allopurinol/Febuxostat

Microalbuminuria in a Diabetic: >30 mg/24 hr to 300 mg/g in 24 hrs (Spot test)


Macro albuminuria in a Diabetic: >300 mg/24 hr

Proteinuria: 1+, 2+, 3+ --> Nephritic Syndrome (usually)


Proteinuria: 4+ 5+ --> Nephrotic Syndrome
-----------------------------------------------------------------------------------------------------------------------

Episode 289 Ultra HY Alcoholic Podcast


Note: Material in this outline may be slightly out of order compared to the podcast, notes were
graciously provided by Divine Intervention from an anonymous contributor.
● Classic vignette for delirium tremens? Pt who needs surgery following trauma
accident then develops hallucinations
○ Tx for alcohol withdrawal? Benzo’s
○ PPx for alcohol withdrawal? Chlordiazepoxide (long-acting benzo’s)

● Tx for alcohol use disorder? Naltrexone (or acamprosate) + 12-step program (Alcoholics
Anonymous)

● Where is EtOH absorbed? Stomach and small intestine (duodenum)

● Where is EtOH metabolized? Liver


○ Pathway: Alcohol → acetaldehyde → acetic acid.
■ CYP2E1 metabolizes alcohol
■ Alcohol undergoes zero-order elimination → so, EtOH is eliminated in
fixed amount per unit of time
● HY Note: Phenytoin, ethanol, and aspirin undergo zero-order elimination (“PEA =
0-order”)

● Liver finding a/w chronic alcoholism? Alcoholic fatty liver disease aka alcoholic
steatohepatitis
○ Mechanism? Chronic EtOH results in ↑ acetic acid → ↑acetic acid is converted
into ↑ acetyl CoA → acetyl CoA is a substrate for triglyceride synthesis → ↑ TG’s

● Acetaldehyde dehydrogenase is found in stomach and liver


○ Women have less acetaldehyde dehydrogenase enzyme in gastric lining → so,
women have higher EtOH bioavailability than men

● HY: Know the enzymes that metabolize alcohol!!


○ Men vs women differences!

● Etiology of high-anion gap metabolic acidosis a/w chronic alcoholism? Lactic acidosis
○ Mechanism? See below

● Glucose derangement a/w chronic alcoholism? Hypoglycemia


○ Mechanism? See below

● Does chronic alcoholism promote fat synthesis or fat breakdown? ↑ Fat synthesis
○ Mechanism? See below

● Mechanism of lactic acidosis + hypoglycemia + ↑ fat synthesis a/w chronic alcoholism?


○ Normally, NADH signals to liver cells that the body has ample energy

○ Excess alcohol metabolism results in excess NADH generation → excess NADH


falsely tricks the body into thinking there is amply energy → this results in…
■ … Excess conversion of pyruvate into lactate → this leads to lactic
acidosis
● Note: For compensatory mechanism, pick “decreased bicarb”

■ …Decreased gluconeogenesis → results in hypoglycemia

■ … Decreased fat breakdown → results in fat accumulation at liver


(alcoholic fatty liver disease!)

● Note: What other enzyme reactions are upregulated d/t chronic alcohol metabolism?
○ Acetoacetate → beta-hydroxybutyrate
■ Results in ketoacidosis
○ Dihydroxyacetone phosphate → glycerol-3-phosphate
■ Glycerol-3-phosphate is a substrate for triglyceride synthesis → so, it
further contributes to alcoholic steatohepatitis!

● AST:ALT ratio in alcoholic hepatitis? > 2:1 AST:ALT ratio


○ Mechanism? Alcohol kills mitochondria and AST is a mitochondrial enzyme → so
death of mitochondria releases tons of AST

● HY enzyme elevation in alcoholics? ↑ GGT!


○ Mechanism? Chronic alcoholics have chronically ↑ lipid metabolism → lipid
metabolism occurs on smooth endoplasmic reticulum → SER contains GGT
enzyme → thus, GGT is elevated

SEQUELAE OF CHRONIC ALCOHOLISM

● Gout and Uric Acid kidney stones


○ Mechanism? Lactic acid + beta-hydroxybutyrate are excreted at kidneys by same
transporters that excrete uric acid → so ↑↑ lactic acid and ↑↑
beta-hydroxybutyrate outcompete uric acid for excretion → results in
under-excretion of uric acid → leads to gout and uric acid nephrolithiasis

● Megaloblastic anemia (2/2 folate deficiency)


○ Mechanism? Alcohol inhbits reabsorption of folate at gut
○ Lab findings? MCV > 100 + ↑ Homocysteine + normal methylmalonic acid

● Rhabdomyolysis
○ Mechanism? Alcohol is toxic to muscle
○ Classic vignette? Person drank alcohol + “found down” by friend next day/many
hours later
■ “Alcoholic passes out for hours → so, body muscles not being used → so
muscle cells die
○ Specific kidney injury a/w rhabdomyolysis? ATN (intrinsic AKI)
■ Released myoglobin → causes acute tubular necrosis.
○ UA findings? 3+ blood but no RBC’s
○ Classic sxs of rhabdomyolysis? Dark red urine + ↑ SCr + hemoglobinuria + no
RBC’s
■ Note: vignette will say “positive hemoglobinuria” b/c urinalysis cannot
distinguish myoglobin vs hemoglobin – this is why “no RBCs on
microscopy” is important to DDx myoglobinuria (i.e. rhabdomyolysis) vs
hemoglobinuria

● Porphyria cutanea tarda


○ Classic presentation? Chronic blistering skin in sun-exposed areas + elevated
AST & ALT + brown/tea-colored urine + elevated uroporphyrinogen
○ Mechanism? Chronic alcoholism downregulates UROD enzyme → leads to
defective heme synthesis pathway → accumulation of uroporphyrinogen
○ Enzyme deficiency? UROD deficiency

● Note: What diseases are a/w porphyria cutanea tarda?


○ Chronic alcoholism
○ HCV
○ HIV

● Acute intermittent porphyria


○ Classic presentation? Abdominal pain + seizures/hallucinations/disorientation
+ red-purple urine (port-wine colored urine) + elevated ALA & porphobilinogen
■ aka abdominal pain + neuro-psychiatric sxs!
○ Mechanism? Chronic alcoholism downregulates PBGD enzyme → leads to
defective heme synthesis pathway → accumulation of ALA and porphobilinogen

EFFECT OF ALCOHOLISM ON CYP P450 ENZYME

● Effect of acute alcoholism on CYP P450 enzyme activity? ↓ CYP P450 activity
○ Drugs metabolized by CYP 450 accumulate → can lead to drug toxicity!

● Effect of chronic alcoholism on CYP P450 enzyme? ↑ CYP450 activity


○ Drugs metabolized CYP 450 are metabolized quickly

● Tx for chronic alcoholism? Naltrexone and acamprosate


○ If refractory → Disulfiram

● Note: What abx is a/w disulfiram-like reaction? Metronidazole


MORE SEQUELAE OF CHRONIC ALCOHOLISM

● Infertility
○ Alcoholism can cause infertility including erectile dysfunction.
○ Any cells that have ↑ mitochondrial content are destroyed by chronic alcoholism.
○ Men develop smaller testes + ↓ testosterone levels + ↓ libido

● What 2 birth defects are a/w fetal alcohol syndrome? Abnormal philtrum + VSD

● Esophageal Tears
○ Mallory-Weiss Tear → Alcoholic + coughing up blood + HDS
■ NBSIM? Endoscopy
○ Boerhaave syndrome → Alcoholic + coughing up blood + HDUS +
subcutaneous emphysema or sxs of shock or pneumomediastinum or
pneumopericardium →
■ NSBIM? Water-soluble contrast enema (or gastrography) + immediate
surgery!
● Do not use barium!
■ Note: Boorhaave syndrome is also called “esophageal rupture” on the
USMLE!

● Liver failure (2/2 portal HTN)


○ Mechanism? 2/2 portal hypertension
○ Esophageal finding a/w liver failure? Esophageal varices

● Acute Pancreatitis
○ Mechanism? Damage to acinar cells → release of pancreatic enzymes

● Chronic Pancreatitis
○ Mechanism? Pancreas is calcified
○ Endocrine sequelae? Diabetes Mellitus d/t ↓ insulin levels 2/2 pancreatic cell
damage
○ GI sequelae? Fat malabsorption with oily stool.
■ NBSIM? Pancreatic enzyme supplementation
● Do not pick “pancreatic resection”

● Cerebellar atrophy
○ Mechanism? Atrophy of Purkinje fibers + cerebral cortex 2/2 mitochondrial death
● Subdural Hematoma
○ Mechanism? Shearing of bridging veins
■ brain is atrophied aka smaller brain → so, brain dances around in skull →
this increases susceptibility to shearing of bridging veins

● Wernicke-Korsakoff Syndrome
○ Associated vitamin deficiency? Thiamine deficiency aka B1 deficiency
○ Mechanism? Several mechanisms:
■ EtOH destroys enzymes that normally convert thiamine to its co-factor
form → so, biochemical pathways that use the thiamine co-factor stop
running
■ Thiamine normally stored in liver, but chronic alcoholism forces liver to
store fat (which is not its primary job – that is the job of adipose tissue) →
so, the body is depleted of thiamine stores
○ Wernicke encephalopathy = confusion + ophthalmoplegia (e.g. nystagmus) +
ataxia
■ Reversible or irreversible? Reversible
■ Tx? First thiamine then glucose (give together!)
● Note: Administer thiamine first, otherwise glucose metabolism will
deplete any remaining thiamine – this is b/c thiamine is a cofactor
for glucose metabolism enzymes
○ Korsakoff Syndrome = amnesia (retrograde or anterograde) + confabulation
(problem with memory source, person agreeing with false statements)
■ Reversible or irreversible? Irreversible
■ Affected brain structure? Mammillary bodies

● Note: Give thiamine then IV glucose to all hypoglycemic pt’s presenting to ED!
○ b/c you may not know who is or is not an alcoholic!

● Note: Etiologies of thiamine deficiency?


○ Chronic alcoholism
○ Hypomagnesemia
○ Gastrectomy
○ Chronic malnourishment (e.g. immigrant on “polished rice” diet, which is not
fortified with B1)
● Cardiac pathology a/w thiamine deficiency? Wet beri-beri (form of dilated ischemic
cardiomyopathy)
○ Mechanism? B1 deficiency results in ↓ ATP production (b/c ATP synthesis
requires thiamine) → cardiac muscle normally needs tons of ATP → so, cardiac
muscle does not have enough ATP to function properly

● What electrolyte is required for proper thiamine function? Magnesium


○ Hypomagnesemia must be corrected for B1 to work!

● What electrolyte abnormalities are a/w hypomagnesemia?


○ Hypocalcemia (calcium repletion will not work if magnesium is low)
● EKG finding? Prolonged QT
○ Hypokalemia (potassium repletion will not work if magnesium is low)
● EKG finding? Prolonged QT

● Tx for methanol poisoning? Fomepizole


○ Mechanism? Inhibits alcohol dehydrogenase → this prevents formation of formic
acid/formaldehyde

-----------------------------------------------------------------------------------------------------------------------

Episode 304 Floridly HY Ortho/Trauma Podcast Part 2


High-speed MVC + BP is 60 mmHg over palpable + widened mediastinum --> aortic transection
(ligamentum arteriosum torn)
Next best step? Surgery exploration
Pathophys? Bleeding into thoracic cavity
NOTE: pt can have trouble speaking d/t left laryngeal nerve

Chronic HTN + widened mediastinum + left pleural effusion--> aortic dissection


A/w RCA infarct--> ST elevations II, III, avF

Government agency worker + works with sheep or in textile factory + widened mediastinum --> anthrax

Medical pathologies a/w aortic dissection


Connective tissue diseases (e.g., EDS)
Tertiary syphilis
Vaso vasorum supplies the walls of aorta

Child, high speed MVA--> most likely struck on RIGHT side


USA- driving on RIGHT
Waddell's Triad: femoral shaft fracture + intra-abdominal or thoracic injury + contralateral head
injury
Child will be struck by bumper--> child thrown on hood of car (causes intra-abdominal/thoracic
injury), bouncing off the hood leads to head being hit (contralateral head injury)
First: Non-contrast CT to see any bleeding

Penetrating injury (e.g., gunshot wound)--> what do you do?


Perform surgical exploration
Stab injury--> potential to form arterio-venous fistulas
Long term: high output heart failure
Pathophys? Fistula creations causes blood to go from artery to vein (cutting off arterioles
(resistance vessels!) and capillaries); SVR goes down, CO goes up
Causes tissue hypoxia
At baseline, the CO is going to be high (all the time)- heart is working hard--> will begin to
decompensate (demand ischemia)

Pt with high output CF; MVO2 will be increased (same as in septic shock, also a potential cause of high
output CF)
Oxygen tension of blood returning to right atrium (MVO2) is higher
Refer Episode 273 for more info on this

Severe trauma--> mistakenly cut off finger/other appendage


Next best step: wrap in moist gauze, put in plastic bag, and put bag on ice
NOTE: for teeth, don’t scrub it because it will de-mineralize, rather, you can put it in milk

Trauma/MVA--> on PE has upper spinal tenderness or weakness


Worry about C-spine problems
Next best step: cervical collar + lateral neck X ray (pick this first before CT in answer choices)

Intubations
Classic indications for intubations?
GCS < 8
Gurgling sounds on auscultation
Expanding hematoma in neck

Pt with intubation few mins ago + becomes hypoxic, O2 not rising appropriately + worsening abdominal
rigidity and diameter
Think: Esophageal intubation
Confirm with CXR
Next best step: take out and re-intubate

Pt with 3 day old intubation + ventilator alarm triggered + higher than normal pressure + CXR: left white
out lung
Think: right main stem bronchus intubation (left main bronchus is wider, shorter, more vertical)
Biggest risk factor: emergency intubation
CXR LEFT side because atelectasis--> air not flowing to left side; blood is still flowing to left, which
will reabsorb all the gas in left lung, but it is not replenished--> resorption atelectasis
Compression atelectasis?
Pneumothorax
Pleural effusion

Intubation + After few mins, harder to intubate + Leukocytosis, hyperkalemia


Think: Malignant hyperthermia
A/w with genetic mutation in dihydropyridine/ ryanodine receptors (ryanodine more
common)
Autosomal dominant
Tx: Dantrolene (prevents Ca2+ from being released from sarcoplasmic reticulum)
Sedate first before neuromuscular blockade
Key associations:
Hyperkalemia
Do:
Cardiac monitor
Calcium gluconate
Insulin with glucose
AKI
Do:
Aggressive hydration

If you are having trouble with intubation after multiple attempts + unsuccessful
Next best step: perform cricothyroidotomy (surgical airway)

Intubated for 2-3 weeks + mild resp difficulty several months later + auscultation of chest= stridor
Think: tracheal stenosis
Biggest risk factor: prolonged intubation

Pt has penetrating injury/ stab injury--> what do you do with the penetrating object?
DO NOT pull out
Remove in controlled conditions (e.g., theater)

When should you NEVER send a pt for CT?


-Unstable pt

Bone vascular nerve


Pt with trauma involved in accident --> extremities involved, bone dislocation/fracture/etc.--> XYZ pulse
not palpable, no sensation in area
Always fix the BONE problem FIRST
Quickly done (reduce dislocation, etc.)
Vascular second
Nerve third
-----------------------------------------------------------------------------------------------------------------------
Episode 305 Upper Limbs Rapid Review 1
Clinical contexts & Clinical Scenarios for upper limb anatomy

Only bony bony connection between AXIAL and APPENDICULAR skeleton?


Clavicle (manubrium of sternum to Acromion of Scapula)

Person with ROTATOR cuff tear, most commonly torn Tendon?


Supraspinatus

Contents of rotator Cuff: SITS


Supraspinatus, Infraspinatus, Teres minor, Subscapularis

Patient with hypertrophy of certain NECK muscles + Brachial plexus symptoms


Anterior and Middle Scalene muscles affected

Patient is unable to extend wrist (wrist drop): Radial Nerve Injury


Radial nerve injury in Borderline Personality Disorder patients who slash their wrists; Lead
Poisoning —> neuropathy

Nerve that innervates ANTERIOR compartment of arm(shoulder to elbow)?


MUSCULOCUTANEOUS nerve

3 HY muscles innervated by musculocutaneous: BBC


Biceps, Brachioradialis, Coracobracialis

Cord of Brachial Plexus that creates Musculocutaneous nerve? LATERAL cord

Nerve that innervates the POSTERIOR compartment of the arm (extension work)? RADIAL
nerve

Anterior/Posterior; Flexion/Extension; MR (musculocutaneous/radial)

Anconeus muscle: Innervated by Radial nerve


Triceps muscle: innervated by Radial Nerve

Blood supply from Subclavian artery to Ulnar artery:


Subclavian artery —> Axillary Artery (lateral border of rib 1) —>Axillary Artery (inferior border of
teres major muscle) —> Brachial Artery —> Radial and Ulnar Artery

HY blood vessel that travels with radial Nerve: Deep Brachial Artery (Profunda Brachii)

Cord of brachial plexus supplied by Axillary nerve? POSTERIOR cord


Other nerve supplied by the posterior cord? Radial Nerve
Cords of brachial plexus that supply the MEDIAN nerve? Lateral and Medial Cord

Cord of brachial plexus that supplies ULNAR nerve? Medial Cord

Nerve and Artery associated with fracture of surgical neck of Humerus?


Axillary Nerve + Posterior Circumflex Humeral Artery

Injury of Axillary Nerve: ANTERIOR shoulder dislocation

Most common type of shoulder dislocation? Anterior


Point of weakness: Move down & forward —> Anterior

Causes of POSTERIOR shoulder dislocation? Electricity


Struck by lightning, Seizures, Electrocution

Nerve and Artery associated with a SPIRAL/midshaft fracture of humerus?


Radial Nerve + Profunda Brachii Artery

Nerve and Artery associated with a supracondylar fracture of humerus?


Median Nerve + Brachial Artery

Breast surgery/mastectomy patient with bulges on upper part of back? Winged scapula
Common complication of breast surgery —> Long thoracic nerve palsy

Long thoracic nerve supplies serratus anterior muscle


SALT: Serratus Anterior, Long Thoracic

Arm Abduction
0-15 degrees: Supraspinatus Muscle (Rotator cuff injury —> difficulty initiating abduction;
empty can test/NEER test)
15-90 degrees: Deltoid Muscle (Issue with Axillary nerve prevents abduction to 90)

Boundaries of Quadrangular spac (Transmits Axillary nerve and Posterior Circumflex Humeral
Artery)
Superior Border: Subscapularis & Teres Minor Muscle
Inferior Border: Teres Major Muscle
Medial Border: Long head of Triceps Brachii
Lateral Border: Surgical neck of Humerus

Key high yield spaces Bordered by triceps (Quadrangular space, Triangular space, Triangular
interval)

Most common type of shoulder dislocation? ANTERIOR (90% of shoulder dislocations)


Most parts of shoulder joint are held by ROTATOR CUFF (inadequate reinforcement for anterior
shoulder joint)

Key anastamoses:
Scapular Circumflex Artery <—> Suprascapular Artery (Blood supply to Supraspinatus &
infraspinatus muscles)

Subclavian Artery —> Thyrocervical Trunk —> Suprascapular Artery (crosses over the top of
scapula; supply supraspinatus and infraspinatus muscles)

Muscle that does MOST things for the arm? Deltoid Muscle
FEAR (Flexion, Extension, Abduction, Rotation-medial & lateral)

Blood and Nerve Supply of Deltoid Muscle?


Axillary Nerve & Posterior Circumflex Humeral Artery

Muscle that ADducts and medially rotates arm? TERES MAJOR


Supplied by lower Subscapular nerve

Nerve Supply of Teres Minor? Axillary Nerve


ADT: Axillary nerve supplies Deltoid Teres minor

Muscle that laterally rotates the arm innervated by Suprascapular nerve? Infraspinatus Muscle

Teres minor laterally rotates arm, innervated by Axillary nerve

Lateral arm rotators: DIT (Deltoid, Infraspinatus, Teres minor)

Muscle that medially rotates arm and ADducts arm: Subscapularis


Innervated by upper and lower Subscapular nerves

Deltoids ABduct arm and medially rotate (vs Subscapularis ADduct arm and medially rotate)
Innervated by Axillary nerve

Nerve supply of Pectoralis Major? Medial and Lateral Pectoral Nerves


Radical Mastectomy —> injure these nerves

3 HY Humeral functions of Pectoralis Major?


Flexion, ADduction, and Medial rotation Humerus (FAM)

Muscle that depresses and protracts scapula: Pectoralis Minor


Supplied by Medial Pectoral Nerve

Blood and Nerve supply of Serratus Anterior (SALT)?


Long Thoracic Nerve & Lateral Thoracic Artery

2 HY flexors at shoulder joint? Deltoid & Pectoralis Major

3HY extensors at the shoulder joint? Deltoid, Teres Major, Latismus Dorsii

3 HY Adductors at shoulder joint? Pectoralis Major, Teres Major, Latismus Dorsii

2 HY abductors at the shoulder joint? Supraspinatus, Deltoid

5 HY medial rotators at shoulder joint? Pectoralis, Teres Major, Latismus Dorsii, Deltoid,
Subscapularis

3 HY lateral rotators at Shoulder Joint? DIT: Deltoid, Infraspinatus, Teres Minor


-----------------------------------------------------------------------------------------------------------------------

Episode 308 Floridly High Yield NBME Cortisol


Cortisol excess: high weight, insulin resistance, buffalo hump, decrease in bone mineral density
( compression fractures ), metabolic alkalosis electrolyte levels: hypokalemia, but sodium levels
may be normal skin hyperpigmentation ( should not be your clue that you are banking on to say
that he has hypercortisolism, it just indicates high ACTH levels ( can occur in adrenal
insufficiency also )
ACTH comes from POMC ( pro-opio-melanocortin ) opioid+MSH+ACTH

Patients will also have hyperkalemia because corticosteroids have a mild mineralocorticoid
effect, can bind mineralocorticoid receptors
This is the reason why the body has 3-beta-hydroxysteroid dehydrogenase 2 ( which is inhibited
by glycyrrhetinic acid found in licorice )
Normal pathway:
3BHSD2 converts cortisol to cortisone.
Cortisol is active on MR whereas cortisone is not active on MR.
Body takes advantage of that pathway to deal with the cortisol problem because you don't want
to have a mineralocorticoid like state when your cortisol is high
Patient takes a ton of licorice: develops a Conn syndrome like presentation: high BP,
hypokalemia, metabolic alkalosis.
In this state, high levels of cortisol are built up, and not converted to cortisone because of
inhibition of 3BHSD2, thus exerting mineralocorticoid like effect by acting on the
mineralocorticoid receptors

Cortisol deficiency: hyponatremia (excess ADH secretion), hypotension ( cortisol has a


permissive effect on the sympathetic nervous system, has a big effect on controlling peripheral
vascular resistance, as a compensatory mechanism whenever cortisol is low, it leads to
production of high amounts of ADH ( makes sense because ADH helps to retain fluid which may
help in maintaining blood pressure.
On NBME exams, hypocortisolism is almost always associated with low blood pressure for the
reason described above
Due to high ADH, lot of free water is being absorbed by the nephron causing hyponatremia
Hypocortisolism is a bonafide cause for SIADH
Hypocortisolism also causes mild hyperkalemia because cortisol has some activity on the
mineralocorticoid receptors which is now lost, it also causes metabolic acidosis.

When there is cortisol excess, glucose tends to be high because cortisol is a diabetogenic
hormone
In the same way when cortisol is low, glucose tends to be low.

On NBME exams, hypocortisolism is almost always associated with low glucose levels.

How to differentiate HHS/DKA from adrenal insufficiency?


Both have similar presentations eg: hypotension, hyperkalemia ( although in DKA/HHS total
body potassium is low) etc.
The way to differentiate is by glucose level which will be really high in a patient of DKA, HHS,
and very low in a person who has addisonian crisis.

Causes of cortisol excess


● Exogenous steroid consumption ( most common )
○ For autoimmune diseases
○ For chronic asthma management
○ For chronic COPD management
● Primary hypercortisolism ( primary means problem is in the adrenal gland, for eg an
adenoma which is secreting lot of cortisol )
● Secondary adrenal excess
○ Cushing’s disease: ACTH producing pituitary adenoma which stimulates the
adrenal cortex ( not the adrenal medulla ). Adrenal cortex is derived from
mesoderm whereas adrenal medulla is derived from the neural crest. ACTH is
trophic only to the adrenal cortex.
■ Cushing’s syndrome means hypercortisolism whereas cushing’s disease
is due to ACTH producing pituitary adenoma
■ Cushing’s disease is a cause of Cushing’s syndrome
○ Small cell lung carcinoma producing ACTH
● Tertiary hypercortisolism ( very rare )
○ Hypothalamus producing excess amounts of Corticotropin releasing hormone
which stimulates ACTH and cortisol release subsequently

If you see the patient has symptoms of hypercortisolism ( buffalo hump, obese, hyperglycemia )
Investigations
● Document person has hypercortisolism ( 3 ways )
○ Increased 24 hour urinary cortisol
○ Late night salivary cortisol
■ Late at night you’re supposed to be relaxed, not to worry about stuff, so
cortisol is supposed to be low at nigh. If your cortisol is high at night, that
makes you a potential candidate for hypercortisolism
○ Low dose dexamethasone suppression test
■ Normal: after giving low dose dexamethasone at night, the next morning
their cortisol is supposed to be low
■ If you notice that the next morning their cortisol is elevated ( failed to
suppress ), this indicates hypercortisolism
● Measure levels of ACTH ( some cases have high ACTH, some have low ACTH ) . This is
going to help you to find out the cause of ACTH
○ If ACTH levels are low, this means something is suppressing the pituitary gland
to not release ACTH, so the probable cause is person is taking exogenous
corticosteroids or from an adrenal adenoma making ton of cortisol
■ Next step in this circumstance: Abdominal imaging ( Abdominal
CT/MRI, Adrenal CT/MRI )
○ If ACTH levels are high, indicates pituitary adenoma ( Cushing’s disease ) or the
person has another location in the body ( like the lungs: small cell lung cancer )
which is producing lots of ACTH
■ Differentiate these 2 conditions from high dose dexamethasone test
■ Normal tissue will suppress the cortisol release in response to high dose
dexamethasone ( will suppress the anterior pituitary gland adenoma aka
Cushing’s disease )
● Next step: MRI of brain ( NOT CT SCAN )
■ If the cortisol is not suppressed, it indicates ectopic ACTH production
● Lung imagining : Chest Xray, Chest CT scan to find the small cell
lung cancer.
● Treatment of hypercortisolism
○ Steroid synthesis inhibiting agents like ketoconazole
○ Treat the underlying cause if treatable

Causes of hypocortisolism

Primary adrenal insufficiency ( Addison’s disease ) : m/c in the US also known as autoimmune
adrenalitis
Most commonly antibodies are produced against 21 hydroxylase ( necessary for production of
cortisol and aldosterone )
○ Tx is by steroids and fludrocortisone to replace corticosteroids and aldosterone
○ Patients will addison’s disease will also have past medical history of some other
autoimmune disease ( rule of thumb: 90% of NBME questions about an
autoimmune disease will have PMHx of some other autoimmune disease:
Hashimoto’s, vitiligo, pernicious anemia
○ Addison’s disease is commonly associated with Hashimoto’s/Grave’s disease
Type 1 DM ( called the DTA triad: this is an autoimmune polyglandular syndrome
type-2. Associated with AIRE-LG mutations.
○ AIRE is a transcription factor which helps in presentation of endocrine gland
antigens in the thymus, so the immune system develops tolerance
○ When this is absent, there is autoimmune activity against endocrine glands
● Tuberculosis ( most common cause worldwide )
● Neisseria meningitidis ( water house friderichsen syndrome )
● 21 hydroxylase deficiency
○ Most common cause of congenital adrenal hyperplasia
○ Can present in newborn with symptoms of cortisol deficiency
○ Can present in teenager ( antibodies develop later in life ), produces similar
phenotype like CAH
○ Px: hyponatremia, hyperkalemia, normal anion gap metabolic acidosis, skin
hyperpigmentation ( loss of feedback from cortisol on the pituitary gland leading
to excess ACTH production, stimulating melanocytes)

Secondary adrenal insufficiency ( problem in pituitary gland )

● Pituitary adenoma or craniopharyngioma causing destruction of the pituitary gland that


leads to inability of corticotroph cells to produce ACTH

Tertiary adrenal insufficiency ( hypothalamus )

● Hypothalamic tumor
● Prolonged use of exogenous corticosteroids causing atrophy of the CRH producing cells
in hypothalamus and corticotrophs in the anterior pituitary gland

Patients with autoimmune etiology of adrenal insufficiency ( which is also associated with a low
aldosterone state ) also causes type 4 RTA ( hyperkalemic RTA ) : normal anion gap metabolic
acidosis

Hypocortisolism also causes orthostatic hypotension because as discussed earlier cortisol plays
a role in maintaining the peripheral vascular tone

Also causes eosinophilia, because steroids cause eosinophil apoptosis, so in state of low
corticosteroids, eosinophils have an incrreasd half life

Other causes of adrenal insufficiency

● Adrenal-leukodystrophy ( X linked inheritance )


○ Child (boy), loses motor milestones, developmental delay
○ Brain imaging shows diffuse demyelination
○ Problem with peroxisomes, leading to problems with beta oxidation of very long
chain fatty acids
○ We need fatty acids to make myelin that is why there is demyelination
○ Many of the adrenal cortex hormones are steroid hormones and their precursor is
cholesterol and cholesterol is ultimately made from fat, that is why in states of
defective fatty acid oxidation, there is a problem in synthesizing cholesterol and
hence the adrenal insufficiency
○ Die pretty early in life
● Abrupt withdrawal of steroids
○ Person taking steroids for a long time, causing suppression of the
hypothalamo-pituitary-adrenal axis ( HPA axis ): genetic suppression
○ Atrophy of CRH producing neurons in hypothalamus, corticotrophins in pituitary
glands
○ When such patients are exposed to some sort of stress ( trauma, accident, ICU )
they can develop critical illness associated adrenal insufficiency
■ Patient has presented with shock, blood pressure is refractive to
administration of fluids and vasopressors
■ Whenever patient experiences stress , the normal body can release
cortisol, but if there is atrophy of the cells, they are not able to produce
cortisol and patient goes into adrenal crisis
■ Tx: administer a stress dose of steroids to cover for this atrophied HPA
axis organs
■ Person with abetalipoproteinemia, they can't construct those lipoproteins
well, cannot absorb lipids and TG from GI tract, which means the body
cannot make steroids leading to adrenal insufficiency

How to diagnose adrenal insufficiency in exams?

● Stimulation test: Cosyntropin stimulation test ( ACTH analogue )


○ If cortisol fails to rise, that is diagnostic of primary adrenal insufficiency
○ In cases of primary adrenal insufficiency, there is hypotension,so RAAS is going
to be in high gear
○ There will be high renin, high angiotensin-2 but guess what aldosterone is gonna
be low
○ Because when there are antibodies against 21 hydroxylase ( as in the most
common cause of autoimmune adrenalitis ) you literally cannot make aldosterone
○ But if the problem is outside the adrenal gland, aldosterone levels will be high (
for NBME purposes )
● Ultimate treatment is steroids and fludrocortisone
● Hydrocortisone : used for stress dose of corticosteroids ( has both mineralocorticoid and
glucocorticoid effect )
-------------------------------------------------------------------------------------------------------------------------------
Episode 310 Floridly HY Knee Exam and Pathologies

1. 1st inspect (look 1st)


2. 2nd Palpate (feel 2nd)
3. 3rd ROM (move 3rd)

4. VaLgus test
a. Lie supine
b. Flex knee 20 degrees (creating small triangle under knee)
c. Apply LATERAL force to test MCL
i. If a lot of displacement --> rupture or torn MCL

d.
5.
6. ACL
a. Suddenly slow down while running & smashes lateral aspect of knee
b. Ant Drawer sign- tibia moves anteriorly a lot compared to unaffected knee

c. Lachman test – flex knee 20 degrees

i. If both ant drawer sign & lachman given as ans choices= pick LACHMAN
(more sensitive)
d. 2 HY things about ACL tears
i. Usually a/w fracture of lateral tibial plateau
ii. If pt has meniscal tears --> need to assess for ACL tear

7. PCL
a. If tibial dislocation posteriorly
b. Post Drawer sign – tibia moves posteriorly a lot relative to femur
8. Meniscal tears
a. “clicking/catching/locking of knees” + joint line tenderness (medial or lateral)
b. McMurray test for LATERAL meniscus
1. Positive if click/pop/catch at joint line
ii. Lie supine (on back)
iii. Put knees in 90 degree flexion
iv. Use 1 hand to support ant knee- thumb on medial joint line; index on
lateral joint line
v. Other hand on pt heel/foot
1. Apply VARUS (medial) force for Lateral meniscus (VIRUS)
a. VIRUS for lateral = vaRus force (medial) IR= internal
rotation
2. Apply VALGUS (lateral) force for Medial meniscus
c. Apley compression test
i. Place prone & flex knee 90 degrees
ii. One hand- stabilize hit
iii. Other hand-grasp feet & compress foot downward then internally &
externally rotate legs
iv. Positive=a lot of pain

v.
vi.

9. Plumber or gardener pain/redness/swelling RIGHT in front of patella


a. Prepatellar bursitis
i. TX- aspirate & drain

10. Playing sports & heard popping sound, now cant straighten (extend) knee. Pain,swelling,
palpable defect along superior patella
a. Quadriceps rupture

11. BELOW INFERIOR BORDER OF PATELLA


a. Tenderness at medial joint line
i. MCL or medial meniscal tear
b. Tenderness in middle of medial joint line & Tibial tuberosity
i. Pes Anserine bursitis
c. Tenderness over tibial tubercle (tuberosity)
i. Osgood Schlatter AKA traction apophysitis
d. Tenderness at lateral joint line
i. Lateral meniscal OR Lateral collateral ligament tear

12. ABOVE patella


a. Tenderness At lateral femoral condyle
i. Iliotibial band syndrome

13. BACK of patella


a. Pain, swelling behind knee (popliteal fossa)
i. Bakers cyst

14. How do you assess for effusion in knee?


a. “Milking technique” – go to medial portion of knee joint & milk from bottom to
top, then milk laterally top to bottom
i. If medial bulge= knee joint effusion
15. Clarks Test
a. Straighten legs & ask pts to contract quadriceps (tighten thighs), then you press
superior pole of patella to prevent from moving
i. If A LOT of pain while doing this= positive test
b. Patello-femoral pain syndrome

-----------------------------------------------------------------------------------------------------------------------

Episode 313 Floridly HY Hyperkalemia


Normal K levels - 3.5 - 5.0 mEq/L.

An asymptomatic patient with a K level of 7. If a patient presents with a K level of 7, they


SHOULD be having symptoms, but if they’re not, then we probably checked the K levels from a
hemolyzed blood sample, 98% of the K in our bodies rests inside our cells, so a local
breakdown of many many blood cells or muscle cells can lead to a false reading.

Patient misses a dialysis session, what’s causing the hyperkalemia? Renal failure.

Young female, shortness of breath, altered mental status, skin hyperpigmentation,


hyperkalemia, low bicarb - Addison’s disease.
Autoantibodies against 21 hydroxylase. Cannot make aldosterone (absorbs Na through
ENaC channel, causing K to leak through urine).
Aldosterone deficiency - cannot absorb Na and cannot kick K out into the urine.
Normal anion gap metabolic acidosis. Type 4 RTA (low aldosterone).
CAH patient - most common cause is 21 hydroxylase deficiency. Same story above.

Immobile patient with hyperkalemia - muscles are MADE TO MOVE, so when you’re immoblie,
your muscles will start to break down and release K into the blood.

Alcoholic - passed out for many hours, muscles breakdown, K leaks into blood.

Ecstasy can cause hyperkalemia - patients dancing and partying and using their muscles for
6-7h can lead to muscle fatigue.

Patients taking spironolactone or eplerenone - Aldosterone receptor antagonists, so we are in a


hypoaldosterone state and thus hyperkalemia.

Taking ACE Inhibitor - cannot convert ATI -> ATII


cannot trigger release of aldosterone --> hyperkalemia develops.

ARBs - cannot release aldosterone.

NSAIDs - inhibit COX1, COX2, COX2 causes us to release prostacyclin which can make the
kidneys secrete K+ ions into the urine.
Taking NSAIDs and inhibiting COX2 (celecoxib) -> cannot secrete K+ ions into the urine.

HIV pneumocystis patient taking TMP-SMX --> Block ENaC channel of the collecting duct,
cannot displace the K --> accumulates.

Amiloride, triamterene - K sparing diuretics. Blocking the ENaC channel.

Patient taking beta blockers - β-2 receptors increase the Na/K ATPase pump.
Blocking b2 receptors -> low activity of the Na/K ATPase --> K cannot enter the cell -->
build up outside the cell.

Neuroleptic malignant syndrome/malignant hyperthermia - muscles are breaking down ->


hyperkalemia.

Patient took succinylcholine - depolarizing Neuromuscular Blocking Agent --> Na+ rushes into
cell --> K+ displaced.
This is why burn patients shouldn’t be intubated with succinylcholine.
Burn patients have a lot of K released already because their muscles have
broken down, succinylcholine causes more K to be released so we cannot give
them.

Digoxin - inhibits Na/K ATPase pump.

Metabolic acidosis - H+ outside of the cell is high.


They will try and enter the cell through the H+/K+ pump --> More K+ released.

Organ transplant - Calcineurin inhibitors (cyclosporine, tacrolimus) also inhibit the Na+/K+
pump.

Pentamidine (for pneumocystis jiroveci) inhibits Na+/K+ pump.

Patients treated for a hematological malignancy - killing many cells in one swoop. Also knocking
out the kidneys by all the uric acid, cannot effectively remove the K+.

Octreotide - suppresses insulin


Insulin increases the activity of the Na+/K+-ATPase pump.

WHY IS HYPERKALEMIA BAD?

K+ is responsible for repolarization


if K is really high outside, then the gradient for K flowing from inside -> outside WILL NOT BE
THERE
Cells will NOT repolarize
So cells are stuck in a depolarized state --> Over time, patients can go into cardiac
arrest.

ECG - tall peaked T wave.


Na will keep rising -> Wide QRS complex.
Treatment
1. Stabilize the myocardial membrane - Calcium salt. Calcium gluconate. Calcium chloride.
2. Give Insulin + glucose.
3. Give beta 2 agonist - nebulized albuterol.
4. Give sodium bicarb - raise pH of blood, attract H+ outside of the cell and K+ will go
inside.
5. Diuretics - most cause hypokalemia.
6. Kayexalate (Sodium Polystyrene sulfonate)

Most effective intervention is HEMODIALYSIS.

Patient has altered mental status after they exercise and develop hyperkalemia with paralysis -
HYPERKALEMIC PERIODIC PARALYSIS.
AD, chromosome 17.
Na+ channels are very very slow to close, displacing K+ ions.
-----------------------------------------------------------------------------------------------------------------------

Episode 325 Extremely HY Screening Guidelines


In pregnancy
First Appointment Screening
Gestational Diabetes
· Start screening at 24 weeks with 1 hour glucose test and
a. If positive follow with 3-hour glucose tolerance test
HIV
· At first prenatal visit
· Retest for HIV in the Third Trimester.
HBV
· At first prenatal visit
o Hep B Surface Antigen.
Syphilis
· At first visit
Asymptomatic bacteria: Increase risk of preterm labor and delivery. (Nonpregnant asymptomatic not
treated)
· Treated with nitrofurantoin o or 1st/2nd gen cephalosporin
· Test of cure
Group B STREP
35-37 weeks anal and vaginal

Women’s RH status and antibody status (BY INDIRECT COOMBS TEST )


· Rhogam at 28 weeks and second dose within 3 days postpartum

Smokers
· Screen for AAA (Abdominal aortic aneurysm)
o Between ages 65-75
o Only men!!!
o If you have smoked as a male or have a family history of AAA
o With an ultrasound
o If more than 5.5 cm you need intervention (endovascular repair of some sort)
· Screen for Osteoporosis
o Screen in woman
o Normally you start screening with DEXA scan at age 65 for general population.
o But in smokers you can start screening for osteoporosis at younger age
o You can also screen for osteoporosis at less than 65 if:
§ Very thin like with Anorexia Nervosa
§ Premature ovarian failure
· Lung Cancer
o You need to be between ages 50-80
o Have a 20 or more pack-year history
o Currently smoking or have quit less than 15 years ago.
o Screen with a low dose CT scan and you only do it 3 times every year
§ If you screen 3 times year after year and it is negative you can stop

Special Genetic Diseases


Familial Hypercholesterolemia

· People get MI at age 25


· People have LDL receptor mutation generally
· Remember rule of 3
o Start checking LDL cholesterol at age of 3, 9 and 18
o You will continue to screen but not tested on NBME

Inflammatory Bowel Disease (Both of them)


· You start screening for colon cancer 8 years after making initial diagnosis.
o Continue screening every 1-3 years after that
· You have a small exception of rule:
o If they are diagnosed with primary sclerosing cholangitis they need to be screened
for colon cancer at the time of diagnosis.
§ Then you continue screening every 1-2 years

Lynch Syndrome

Px tend to have microsatellite instability problems

· Start colon cancer screening at age of 20


o Continue every 1-2 years
o Make sure you go straight to COLONOSCOPY
o You can give aspirin as a prophylaxis!!!
· They have other cancers as well
o Start screening endometrial cancer at age 30
§ Continue every 1-2 years
§ Endometrial biopsy or sampling
§ You offer a prophylactic (THBSO) total hysterectomy with bilateral
salpingo-oophorectomy after child bearing due to astronomic risk of
endometrial cancer and ovarian cancer

Patients with MEN 2A or 2B

· Its not a matter of if but when will you get a medullary thyroid cancer
· You offer a prophylactic thyroidectomy !!!

Familial Adenomatous Polyposis

APC gene mutation-> Polyps -> Kras mutation-> p53 mutation-> Adenocarcinoma

· Start colon cancer screening at age 10 years


· Continue every year

BRCA Mutations

Breast cancer screening guidelines


· Between ages 25-29 you deserve an annual breast mammogram
· If you are over the age of 30 you deserve
o Annual breast MRI and annual mammography

Risky Sexual Group


· Individuals with HIV
· Men that have sex with men
· Sex workers
· IV drug users

Screening guidelines

· If you are between ages 15-64


· You deserve annual HIV
· Screen for syphilis every three months
· Screen for Chlamydia and gonorrhea
o You only screen women, DO NOT SCREEN MEN!
o You screen higher risk patients annually no matter the age
o ALL patients (not only high risk) younger than 24 need annual testing
o Non risky patients don’t need to be screened for these two annually after age of 24
· Screen for Hep B surface antigen every year
o Also screen the antivaxxers every year for hep b

Metabolic Diseases
Hypertension

· Screening is started at age 18 through 40 every 3-5 years


· After age 40 you screen every 1-2 years
o Prevalence and incidence increases dramatically over age 40
o Guideline will change due to increase of burden of cardiovascular disease

Diabetes

· Obese individuals between age 40-70 years old


o Screen every 6 months
o Fasting blood glucose
o Oral glucose tolerance test
o HbA1c
· Hypertension over 135/80 and over age 45
o Needs screening every 6 months!!!!

Osteoporosis
· Screening at age 65
· We only screen Women
· DEXA scan looking at t score
o -2.5 or less
o Needs treatment

Hyperlipidemia

There are three screening guidelines separated into groups:

Everyone gets initial screening at age 20

For metabolic disease the screening guidelines gets more frequent the older you become.

· Men
o Ages 20-45: Every 5 years
o Ages 45-65: Every 1 to 2 years
o Over age 65: Every single year
· Women
o Risk increases more after menopause
o Ages 20-55 every 5 years
o Ages 55-65: Every 1-2 years
o Older than 65: Every years
· Diabetics
o You screen every year, the end!
o If LDL is 70mg/dl or higher they will be placed on a high intensity statin like
rosuvostatin or atorvastatin.

Cancer Screening
Colon Cancer

· Start screening at age 50


· Age 45 or 75 or higher might need colon cancer screening
o Screening has been started at an earlier age now so lets say its age 45 to 75.
· Many ways to screen:
o Colonoscopy every 10 years
o Flexible sigmoidoscopy every 5 years
o Flexible sigmoidoscopy every 10 years if you do a Fecal immune test every year
o Fecal Immunotest or fecal occult blood test every year.
o CT colonography every 5 years will start to be implemented.
§ CT scan that has certain protocols that make it highly sensitive for detecting
colon cancer.
· If any of the tests other than colonoscopy show an alteration, the next best step is to DO
A COLONOSCOPY WITH BIOPSY!!!

Breast Cancer
The problem is NBME uses ACS so know both of them, the question stem will probably give you enough
information to know both of them

American Cancer Society

· Mammogram Start at age 40 and then do it every year

USPSTF guidelines

· Mammogram: Ages 50-75 then every 2 years

Cervical Cancer
· Screen ages 21-29 with pap smear every three years
· If you are 30 to 65 years old
o Preferred modality: Pap smear + HPV contesting every 5 years
o Pap smears every three years
o HPV testing every 5 years
· Special Cervical Screening guidelines
o HIV or immunodeficiencies:
§ They get pap smears every year!!!
§ Especially T cell deficiencies like DiGeorge
· Hysterectomy and pap smears:
o If it was done do to benign conditions like leiomyomas: You do NOT NEED TO DUE A
PAP!
o If it was done due to malignant reason (cervical or endometrial cancer): Pap smear of
the vaginal cuff.
o If hysterectomy was done due to cervical cancer you will continue doing pap smears
until 20 years after the surgery
· Stop screening at 65:
o No history of cervical cancer
o No history or severe Pap smear HSIL OR CIN 3
§ If they did then you need to have 3 negatives in a row
§ Or 2 pap smears plus HPV contesting in a row

Children
Patients with Language difficulties:

· Screening hearing with audiometry

All kids between ages 3-5 years old

· Screen for amblyopia-> Lazy eye

Miscellaneous
Women Less than 24 years old: Screen annually for gonorrhea and chlamydia ONLY IN women
Who needs to be screened for Hep B?

Need to be screened every year

· Patients who receive multiple blood transfusions or are constantly on dialysis


· Long term chemotherapy or long-term immunosuppressive therapy due to transplant.
· With hepatitis b surface antigen

All people between ages Hep C 18-79 once

Screening for depression

· Easy questionnaire:
o PHQ-9
o Beck depression inventory

-------------------------------------------------------------------------------------------------------------------------------

Episode 339 HY Pulmonary Embolism


Pulmonary Embolism (PE): Something that happens when you have occlusion of Pulmonary
Artery
Happens frequently in Ptx with DVT (Blood clot in lower extremity veins, commonly in
peroneal vein --> travels through inferior vena cava --> pulmonary arteries --> obstructs right
heart drainage --> right heart failure)

Pulmonary artery pathophysiologic consequences:


Right Ventricle does not drain properly, SHOCK sets in (depends on size of PE)
Adequate ventilation, inadequate perfusion (V/Q mismatch)
A-a Gradient increases (oxygen to alveoli just fine, perfusion cannot come to capillaries
well; PAO2 fine, PaO2 decreases)
Hyperventilation --> Respiratory Alkalosis (CO2 decreases)

Risk Factors for PE: VIRCHOW’S TRIAD (Stasis, Hypercoagulability, Endothelial dysfunction)
Bed-bound (Stasis), Pregnancy (stasis, hypercoagulability), Long plane/train ride without
moving (stasis),
Factor V Leiden (hypercoagulable state), Antithrombin III deficiency (genetic/acquired cause i.e.
nephrotic syndrome),
Puncture Blood vessel (endothelial dysfunction)

Pulmonary artery occlusion causes:


Fat emboli (orthopedic procedure/recent fracture)
Yellow marrow travels to pulmonary artery
Air embolism (vascular procedure (central line))
Amniotic Fluid Emboli (3rd trimester, IUFD, abortion)

Pleural effusion associated with PE: EXUDATIVE effusion (violate 1 or more of Light’s criteria)
Inflammatory cascade --> inflammatory markers --> increase vascular permeability

Time Frame of PE: ACUTE condition


IF recurrent PEs --> right heart dysfunction --> CTPEH (chronic thromboembolic
pulmonary hypertension)
Short of Breath, Tachycardia (EKG: sinus tachycardia; S1Q1T3)

If you suspect PE: Criteria? Not that important for Exam Q


Low risk? Or High Risk?
Over 35, Smoke, and on OCP --> HIGH RISK (most likely PE)
College athlete, normal BMI, nonsmoker, not hypoxic --> LOW RISK
Low risk --> D-Dimer
If elevated --> CT angiogram of Chest
If normal --> rule out PE
High risk --> straight to CT angiogram of chest
If HIGH risk with negative D-Dimer --> CT angiogram of chest anyways
Pregnant woman suspected of PE: Do a V/Q scan
Positive --> treat for PE
Negative --> CT angiogram of chest
Renal dysfunction: NO CT angiogram of chest
V/Q scan (ventilation/perfusion scan)
Nuclear medicine scan that tests ventilation of lungs, then the perfusion of lungs
Ventilation does NOT match up with perfusion --> most likely a PE

Patient with suspected PE and CT angiogram, V/Q scan, D-dimer NOT answer choices…
Pick Duplex ultrasound of Lower Extremities

Gold Standard test for diagnosing PE: Pulmonary Angiogram (NOT CT angiogram)

Patient has PE: Treatment --> IV HEPARIN


LMWH (all other patients)
Unfractionated Heparin (only in patients with renal dysfunction)
tPA (few situations when it should be picked)
Hemodynamically unstable with PE --> give tPA
Clear definitive signs of Right Ventricular failure (Echo shows elevated RV
distension) --> give tPA

Patients who cannot get Heparin/tPA: recent surgery (bleeding/neurosurgery)


Only option --> embolectomy
Person with DVT with a contraindication (recent surgery, history of bleeding) --> do not give
Heparin for DVT; Place an IVC (inferior vena cava) filter

Long-term therapy for post-DVT: Anticoagulation therapy for 3 months MINIMUM


Anticoagulants: DOACs (Factor X inhibitors, Factor II Inhibitors)
Factor X inhibitors: Rivaroxaban, Apixaban
Factor II inhibitors: Dabigatran, Lepriduin, Bivalirudin, Argatroban
Certain circumstances: Warfarin

Cancer patient or Pregnancy with PE: HEPARIN ONLY


-------------------------------------------------------------------------------------------------------------------------------

Episode 379 A Series of HY “Elderly Vignettes”


79 year old patient who has pyelonephritis, and has to be placed on gentamicin. Compared to a
29 year old, which of the following has to be adjusted? LOWER DOSE of GENTAMICIN
As you get older, GFR DECREASES --> Creatinine Clearance DECREASES --> drugs cannot
be metabolized/cleared as well --> Lower dose for RENALLY CLEARED drugs

Drugs with nephrotoxic effects (renally cleared): Metformin, Lithium

As people get older, BP RISES


Aorta not as elastic as it should be--> cannot expand as well to increased amounts of
blood --> Increases BP

In the elderly, the lungs are like that of emphysema


Similar issues
Lungs are NOT as elastic --> Lungs expand, but do not recoil (similar to an obstructive
lung image) --> Decreased FEV1, FRC Increased, PaO2 decreases (inefficient gas
exchange)

Patient is 75 year old with knee pain, cannot golf in the morning. Has had the problem for a few
months. Dx: OSTEOARTHRITIS
Most Common Joint disease in the elderly

Old patients, on their skin, have ecchymoses and purpura. The patient has decreased skin
turgor. Dx: Loss of skin elasticity
Fibers that contribute to elasticity in skin BREAK DOWN --> blood vessels in skin become
unstable --> rupture easily (petechiae, purpura) --> decreased skin turgor
Wrinkles common in the elderly

Woman in her 60s, comes into the clinic stating that sex with her husband has become more
painful. DX: LOW ESTROGEN STATE
Ovaries become functionally nonexistent (do not work well) --> granulosa cells not
present --> little estrogen present --> no maintenance of vulva and vagina --> no natural
lubrication for intercourse
Tx: Estrogen creams, Estrogen lubricants

Arrows in an OLD FEMALE:


LOW Estrogen, HIGH FSH and HIGH LH, HIGH GnRH (unreliable due to weird secretion
patterns)
Hypergonadotropic Hypogonadism

Male who has lower abdominal pain a few times a month, complains of urinary dribbling.
Patient’s creatinine is slightly elevated. Most likely mechanism behind these problems?
Hyperplasia of the Prostate Gland (BPH)
Prostate gets really big --> squishies the prostatic urethra --> difficult for bladder to drain
--> urinary stasis --> predisposes to infection --> Causes urinary retention --> obstructive
uropathy --> can cause chronic renal failure
BPH management:
Acute (symptomatic) --> catheterization immediately, alpha-1 blocker (terazosin,
prazosin, tamsulosin; open bladder sphincter)
Chronic --> 5-alpha reductase inhibitor (Finasteride, Dutasteride); TURP (transurethral
resection of the prostate)

Do NOT give alpha-1 blocker with OTHER powerful vasodilators (i.e. nitrates for CHF; sildenafil
for ED; Hydralazine for HTN; Amlodipine): may cause ORTHOSTATIC HYPOTENSION

Patient who is placed on alpha-1 blocker what psych meds should you be careful with? (2 meds)
TCAs (anti-HAM symptoms; anti-H1 (sedation), anti-alpha 1 (orthostatic hypotension),
antimuscarinic (anticholinergic toxidrome))
Low-Potency 1st Gen antipsychotics (i.e. chlorpromazine): anti-HAM symptoms

MOA of 5-alpha reductase inhibitors: 5-alpha reductase normally converts testosterone to DHT;
DHT is responsible for growth of prostate
Block 5-alpha reductase --> decrease amount of DHT --> decrease size of prostate by
50% in a few months

As a male gets older, levels of TESTOSTERONE decreases


Notice atrophy of testicles

Most common cause of atrophy of testes: Use of synthetic androgens to bulk up

As a female gets older, levels of testosterone also decreases


Woman’s sex drive DECREASES as they get older (due to ovaries “shriveling up”, low
estrogen and testosterone states)
If a female has a TAHBSO (total abdominal hysterectomy and bilateral salpingo oophorectomy)
Libido will DECREASE (taking away a source of testosterone)

Older person is having trouble reading signs at night, and cannot read at night. Dx: CATARACTS
Cataracts: opacification of the lens
Tx: Lens replacement
Surgery with NO real pre op testing (low-risk surgery)

Older gentleman who is having trouble hearing needs to turn the TV louder during his golf and
basketball matches. Dx: Presbycusis
Presbycusis: Hearing loss; very common in elderly individuals
Pathophys: Hair cells in inner ear not working as well --> Hearing slowly degenerates

In the elderly, the activity of taste buds reduces --> food doesn’t have as much taste as it used
to

Olfactory neurons do NOT work as well --> smell sensation decreases

As a person gets older, they begin to have higher risk of ALL skin cancers
Skin becomes thinner as we age --> UV rays can penetrate much better --> many skin
cancers can occur

Actinic Keratosis (common in the elderly who spend a lot of time outside)
Usually goes away by itself, most patients get better over time
SOME patients progress to Squamous Cell Cancer of the skin
SCC of skin: On histology, keratin pearls present

Elderly can get Basal Cell Carcinoma (due to UV radiation)


Most common skin cancer overall

Elderly can get melanomas (due to UV radiation)

African American with melanoma on the skin, will be found in “weird places” (i.e. under the
nails)
Acral Lentiginous Melanoma

Lentigo Maligna in Caucasians (non-black)

In the elderly, the immune system does not generate cell-mediated immunity well
More susceptible to severe infections
# of CD+ T-cells decreases over time --> cannot deal with viruses and cancers well

As a woman gets older, due to the relative DECREASE in estrogen --> BREAST ATROPHY

The longer time a person has lived, the more time they have had to develop genetic mutations
More malignancies

For individual to have multiple cancers, they most likely have a genetic inherited condition
Lynch Syndrome, Li-Fraumeni Syndrome, BRCA-1 mutation, etc..

69 year old female, the patient complains of lower back pain. Labs reveal normal calcium,
normal ALP, no anemia. Dx: Osteoporotic Fracture
Osteoporosis very common among the elderly, especially in females
Most common site of fracture Is in the vertebra

As a person gets older, the brain gets smaller and smaller


Cerebral atrophy but skull stays the same size --> more likely to have trauma --> veins
that connect brain to meninges can be damaged --> bridging veins pop --> subdural
hematoma /Hemorrhagic stroke
More common in ALCOHOLICS
Alcohol causes decrease in size of cerebrum

Dementias common among the elderly (in order of commonness)


Alzheimer Dementia: beta-amyloid (beta-pleated sheets) plaques, neurofibrillary tangles
Vascular Dementia/Multi infarct Dementia: (1 year one trouble, every 6 months new
troubles); stepwise-decline

As people get older, Type II DM more common


-------------------------------------------------------------------------------------------------------------------------------

Episode 381 Some HY Pharmacology Vignettes


Patient is brought to the ED because a friend found her vomiting. Patient has significant right
upper quadrant pain. Friend saw the patient doing well yesterday, but today she was found
passed out. Patient was vomiting. Labs show ALT and AST significantly elevated. Dx?
Acetaminophen toxicity
Acetaminophen can screw up the liver (used to attempt suicide)
NOT an NSAID (works like an NSAID)
MOA: Inhibits synthesis of prostaglandins (works primarily in hypothalamus)
Decreases sensation of pain, antipyretic, analgesic
Side effect: Liver damage (when used in excess; Acetaminophen converted to NAPQI -->
destruction of centrilobular region of hepatocytes))
Antidote: N-Acetylcysteine (helps regenerate glutathione)

Uses of Acetaminophen:
Child with febrile seizures (with seizures that last less than 15 min), previous history of viral
infection
History of Osteoarthritis (1st line medication; start with exercise and diet prior to meds)
Pain control (if you cannot use a NSAID)
Patient recently started on an anti-hypertensive medication, the patient presents 2-3 weeks
later with significantly elevated creatinine (1 to 3), BUN has increased significantly. Dx?
ACE-Inhibitor Toxicity
ACE-inhibitors are powerful dilators of efferent arterioles (arterial portal system; fed by
afferent, drained by efferent)
Kidney system: Afferent arteriole --> glomerular capillary --> efferent arteriole
MOA: Dilate efferent arteriole --> decrease hydrostatic pressures in glomerular
capillaries --> favor filtration/decreased reabsorption --> Increase creatinine/GFr slightly
Side Effect: Dry cough (due to bradykinin NOT being broken down) (mitigate with ARB)
Bradykinin stimulates coughing, edema
Uses: Hypertension, Hypertension with kidney issue (ESRD), Hypertension with Diabetic
Nephropathy (prevent hyperfiltration injury)
Contraindicated in PREGNANCY
In patients with bilateral renal artery stenosis (kidneys NOT perfused well)
If given an ACE-Inhibitor/ARB --> excess reabsorption --> increase creatinine and BUN
Family who has had a bad reaction to ACE-inhibitors: Dry cough, significant mucosal edema,
abdominal pain. Dx? Hereditary Angioedema
Pathophys: C1-esterase inhibitor deficiency --> excess buildup of bradykinin (only
problematic IF they take an ACE-inhibitor)
Autosomal Dominant inheritance pattern
African Americans and have essential hypertension, no kidney issues. DOC?
Only give ACE-Inhibitor/ARB IF they have a kidney problem

Diabetic Nephropathy: Kimmelstiel-Wilson Lesion/Nodule


Diabetes is one of the MC causes of ESRD (HTN is another MC cause of ESRD)
ACE-Inhibitors improve mortality in Heart Failure
Along with Beta-blockers (metoprolol, bisoprolol, carvedilol), Aldosterone antagonists
(spironolactone, eplerenone), Isosorbide Dinitrate-Hydralazine combo (BiDiL)

If you have wide QRS, cardiac resynchronization can improve mortality

ARNI drugs (valsartan-sacubitril; ARB-neprilysin inhibitor)


Improve survival in heart failure

Normal system: arteriole --> capillary --> vein


Portal systems within the body (when you need to do significant regulation)
Liver: Venous portal system (portal vein --> sinusoidal capillary --> hepatic vein)
Brain/Hypothalamus: Venous portal system
Patient who steps out in the cold, hands/fingers turn blue (cyanotic in digits). Patient also has a
long history of GERD with daily PPI therapy, BP 290/200. Dx? SCLERODERMA RENAL CRISIS
CREST syndrome (Raynaud’s phenomenon, Calcinosis, Esophageal dysmotility,
Telangiectasia
DOC in Scleroderma Renal crisis? ACE-Inhibitor

Pregnant woman with a history of hypertension does NOT receive regular care during
pregnancy. Woman comes in for a prenatal checkup and has oligohydramnios. Cause?
ACE-inhibitor teratogenicity
ACE-inhibitor and ARB are teratogens (cause renal damage in fetus --> no urine
production --> source of amniotic fluid has dried up --> oligohydramnios (no urination
leads to no swallowing leads to no urination)

35 year old female who is hearing voices, and is started on haloperidol. Patient presents with
cogwheeling rigidity, loss of speech, rigidity. Dx? Drug-Induced Parkinsonism
Haloperidol causing drug-induced parkinsonism
Tx: Muscarinic antagonist/Cholinergic receptor blockers (Benztropine,
Trihexyphenidyl), Dopamine Agonist (Bromocriptine, Cabergoline)

Drugs listed above can also be used for parkinsonism (early on)
Benztropine, Trihexyphenidyl; Bromocriptine, Cabergoline

Bromocriptine/Cabergoline (Dopamine agonist) useful to treat prolactinoma


Dopamine = Prolactin Inhibiting Factor (downregulate prolactin production)
Surgery is NOT usually the therapy for a prolactinoma
Dx: MRI with contrast
Symptoms: Bitemporal hemianopsia, headaches, difficult to get pregnant, low libido
(prolactin inhibits GnRH --> lower FSH/LH --> lower estrogen/testosterone production;
Hypogonadotropic Hypogonadism)
-------------------------------------------------------------------------------------------------------------------------------

Episode 404 Floridly HY CXR (Chest X-Ray)


Basics of Chest X-Rays (CXRs)
Trachea (thin black tube in the middle of the throat)
Shifted trachea: Tension pneumothorax
Treatment: Needle Decompression/Needle Thoracostomy (converts the tension
pneumothorax to an open pneumothorax) followed by a chest tube
placement/tube thoracostomy
Lung Apices (very top of lung fields)
Aortic Knob (above the heart)
Enlarged/Widened Aortic Knob: Anthrax, Aortic Dissection
Left border of heart = left ventricle
Size of the Heart (midline to the end of the ribs on the left side --> heart border should NOT
extend more than ½ way)
Size of heart (should be no greater than ½ of the whole size of the thoracic cavity)
Enlarged? Left Ventricular Hypertrophy, Heart Failure
Costophrenic angles (at the bottom)
Should be sharp
Blunt: Fluid in the lungs
Most Anterior section of the heart = Right Ventricle
Most Posterior section of the heart = Left Atrium
TEE (Transesophageal echocardiogram) MORE sensitive than TTE (Transthoracic
echocardiogram)
Border of Heart on the right side: Right Atrium
Border of Heart on the left side: Left Ventricle
In mediastinum, most lines point DOWNWARDS
Lymph nodes, pulmonary blood vessels

4 things to always look at on a CXR:


1. Look at the middle
2. Look at the edges
3. Look at the top
4. Look at the costophrenic angles
If a child is UNDER 10 years old: LOOK AT THE THROAT

TTE vs TEE?
TTE ONLY if TEE is NOT an option (TEE always a better answer than TTE)

Patient comes in with a fever and a productive cough. On chest x ray there is a pneumonia
present. (Any sort of “white” = aka radiodense)
MCC of Pneumonia: S. Pneumonia
One side well circumscribed, an infiltrate --> LOBAR PNEUMONIA
White “stuff” going on all over the lungs --> INTERSTITIAL PNEUMONIA

Typical Pneumonia: Toxic, High Fevers, very sick, Productive Cough, Lobar consolidation (“white”
that is well circumscribed)
Atypical Pneumonia: Indolent course, little to no symptoms, diffuse process on CXR

Hospital-Acquired Pneumonias (Usually S. Aureus or Pseudomonas)


Treat with Vancomycin + Piperacillin & Tazobactam
Or Treat with Vancomycin + Cefepime
OR Vancomycin + Ceftazidime

Community-Acquired Pneumonia: S. Pneumoniae

Which Bug is associated with which Pneumonia?

Person who has been on prolonged steroid therapy (bad asthma)/immunocompromised (HIV)
and has fever, cough and is hypoxic. On Chest X-ray there is a diffuse process. Dx? Pneumocystis
Jirovecii Pneumonia

If patient has a CD4 count < 200, prophylaxis for PJP is given (TMP-SMX)
Very hypoxic patient (PaO2 <70) --> give a corticosteroid with the TMP-SMX

HIV patients can also get a lobar pneumonia (Most likely S. Pneumoniae)

Young patient with very mild symptoms of pneumoniae, on Chest X-ray there is an interstitial
process present. Dx? Mycoplasma Pneumoniae
Alcoholic Patient with Gram (-) organisms in their sputum with pneumonia. Dx? Klebsiella
Pneumonia

Patient with a foul smelling sputum. Dx? Anaerobic Pneumonia

Patient who has pneumonia and is in constant contact with birds/parrots. Dx? Chlamydia
Psittaci Pneumonia

Patient who is from the southwestern part of the USA (i.e. New Mexico) and has a pneumonia.
Dx? Coccidiomycosis

Patient is a farmer who works with farm animals and presents with pneumonia. Dx? Q fever
(Coxiella Burnetii)

Child who is under the age of two and presents with pneumonia. Dx? RSV (Respiratory
Syncytial Virus) Pneumonia

Newborn with pneumonia. Dx? Group B Streptococcus (GBS; Strep Agalactiae)

Patient recently was at a business conference and had a humidifier in his room, presents with
pneumonia. Dx? Legionella

Cystic Fibrosis patient with pneumonia. Cause?


Under 20: S. Aureus
Over 20: P. Aeruginosa

Patient is from Kentucky/Ohio and presents with pneumonia. Dx? Histoplasmosis (Histoplasma
Capsulatum)
Older patient who has been a smoker for a long time, comes in to the doctor with hemoptysis
and has lost weight. On Chest x-ray a density (pulmonary nodule) in the lungs is present. Dx?
Lung Cancer

Patient who is a smoker and has JVD. On Chest X-ray a nodule/mass is visible. Dx? Superior
Vena Cava Syndrome

How to differentiate lung cancers?


Patient has worked in a ship yard or plumbing for a long period of time. On Chest x-ray, the
edges of the chest wall are very thick. Dx? Mesothelioma (or some kind of asbestosis)
Person who has taken a long flight comes to the ED with sudden shortness of breath, dyspnea
and has risk factors (Estrogen containing OCPs, Long car ride/flight, history of malignancy,
smoking, recent surgery, Factor V Leiden, unilateral leg swelling (DVT)). PCO2 is low (respiratory
alkalosis). On Chest CT, there is an “inverted V” present with “black stuff” in it (IV contrast). Dx?
Pulmonary Embolism

Female patient who has shortness of breath, cough, hypercalcemia with a restrictive lung
disease pattern. On chest x-ray there is a diffuse process, on the sides of the heart (lymph
nodes) are enlarged. Dx? Sarcoidosis
Noncaseating granulomas in the lungs (macrophages make excess vitamin D (due to
1-alpha hydroxylase expression) which leads to hypercalcemia)
Tx: Corticosteroids
Patient has a history of COPD OR a tall, thin young male patient in their 20s/30s. On Chest X-ray,
there is a straight line (lung edge visible separate from the chest wall; one side of line very white
and one side very dark) Dx? Simple Pneumothorax
Pathophys: Patient has popped an apical “bleb” --> lung is being squished
Treatment: Chest tube (Tube thoracostomy)
Symptoms: Absent breath sounds, hyperresonance to percussion

Tension Pneumothorax: Deviated trachea common on CXR


Person with penetrating trauma to the chest
OR Patient just got EGD with biopsy
OR Patient got a pneumatic dilation
OR Patient just ruptured esophagus
OR Patient has squamous cell cancer of the esophagus --> esophageal rupture
OR Patient has Booerhave syndrome (Hyperemesis gravidarum, eating disorder).
On Chest Xray, on the tops of the lungs (above the heart) there are black straight/curved lines.
Dx? Pneumomediastinum

Smoking patient has a very large lung field on Chest X-ray. There is a lot of black where the lung
fields should be, lungs are very “long”. Dx? Emphysema
Young smoking patient (smoked for 10 years) and is 40 years old with elevated LFTs. On Chest
X-ray, hyperinflation of the lungs is present. Dx? Panacinar Emphysema (due to alpha-1
antitrypsin deficiency)

Patient is a young child (under 10) with the “steeple sign” or “wine bottle sign”. Dx?
Croup/Laryngotracheobronchitis
Barking, seal like cough; caused by the parainfluenza virus
Person who is an alcoholic/neuromuscular disorder (i.e. ALS, Parkinson’s disease). On chest
x-ray there is an area of darkness with a straight line the horizontal plane (aka an air-fluid level).
Dx? Lung Abscess

Think of Anaerobic bacteria causing lung abscesses


Fusobacterium, Bacterioides Fragilles, Provatella
Tx: Clindamycin (effective against MRSA + Anaerobes)
S. Aureus can also cause lung abscesses

Patient with very blunt costophrenic angles bilaterally on CXR. Dx? CHF exacerbation
Person has been a smoker or has been working at a shipyard for a long period of time and on
Chest X-ray there is a large unilateral pleural effusion. Dx? Malignancy

Top causes of unilateral pleural effusion:


Malignancy (Patient is a smoker who has lost a lot of weight)
TB (patient is a TSA agent, prisoner)
Aortic Dissection (patient has left sided unilateral pleural effusion post MVA)
Will have a widened mediastinum/wide aortic knob

Hemorrhagic Mediastinitis (will also present with a wide mediastinum)


Due to infection with Bacillus Anthracis

Person who has had a very severe illness (acute pancreatitis, emergency surgery, almost
drowned). On chest x-ray the whole lung is white. Dx? ARDS (Acute Respiratory Distress
Syndrome)
Patient will be profoundly hypoxic
Patient who, on CXR, has cephalization (vessels engorging with blood and pointing upward). Dx?
Pulmonary Edema (Decompensated CHF, CHF exacerbation)

Patient has a history of an arrhythmia treated with amiodarone


Patient has a history of RA and has been on long term therapy (i.e. methotrexate)
Patient has recently had a few rounds of chemotherapy with bleomycin/busulfan
On chest x-ray, the lungs are very patchy/white. Dx? Idiopathic Pulmonary Fibrosis
“honeycombing pattern” present

-------------------------------------------------------------------------------------------------------------------------------
Episode 424 HY Rules For Remembering Inheritance Patterns
Autosomal Dominant Disorders

Classic problems: Structural Proteins (collagen, fibrillin, spectrin/ankyrin/band), Receptor of a


membrane (LDL receptor), Gain of function (GoF) mutations (i.e. Achondroplasia GoF mutation
in FGFR3), Hereditary Cancer syndromes (Neurofibromatosis)

Huntington’s disease

X-Linked Dominant (commit to memory)

Fragile X syndrome, Alport Syndrome, Hypophosphatemic Rickett’s disease (vitamin D resistant


Rickett’s disease), Rett Syndrome (mecP2 gene mutation), Lissencephaly, Goltz Syndrome,
Aicardi Syndrome

Mitochondrial Inheritance Pattern (commit to memory)

Look for Ragged-Red Fibers

MELAS (Mitochondrial encephalopathy, lactic acidosis and stroke-like episodes)

LHON (Leber’s hereditary optic neuropathy)

MERRF (Myoclonic Epilepsy with Ragged Red Fibers)

Autosomal Recessive Inheritance

Enzyme-Problems, Mutation in DNA repair gene

SCID (ADA enzyme), Bloom Syndrome, Fanconi Syndrome, Xeroderma Pigmentosum

Sickle Cell Disease, Cystic Fibrosis, Alkaptonuria, Galactosemia, PKU, glycogen storage
diseases, Bartter’s syndrome, Oculocutaneous albinism, Wilson’s Disease

X-linked Recessive Inheritance (share properties with autosomal recessive diseases)

Enzyme-defects

Hunter’s Disease, Lesch-Nyhan Syndrome (deficiency of HGPRT), OTC deficiency, G6PD


deficiency

Immunodeficiency Diseases

X-linked Agammaglobulinemia, (Chronic Granulomatous Disease) CGD, Wiskott-Aldrich


Syndrome
Hemophilias

Hemophilia A/B

Muscular Dystrophies

Duchenne/Becker Muscular Dystrophy


-------------------------------------------------------------------------------------------------------------------------------

Episode 453 Quick Review HY Psych Timelines

PTSD (Post Traumatic Stress Disorder)


Symptoms: A lot of flashbacks, intensified experiences with a history of a bad event
(accident, military, 1st responder)
Symptoms > 1 MONTH
Acute Stress Disorder
Symptoms: A lot of flashbacks, intensified experiences with a history of a bad event
(accident, military, 1st responder)
Symptoms < 1 MONTH

Person with a REALLY bad stressor


Adjustment Disorder: Stressor must have ENDED LESS THAN (<) 6 MONTHS AGO
Symptoms: Depression/Anxiety Lite
WILL NOT meet full criteria for anxiety disorder OR MDD
Symptoms not present for very long

Person who is anxious about MULTIPLE domains of life


Generalized Anxiety Disorder (GAD)
MUST HAVE symptoms > 6 MONTHS

Person has very low mood, anhedonia


Major Depressive Disorder (MDD)
Symptoms (SIG E CAPS) for longer than 2 weeks; 5 out of 9
SIG E CAPS: Sleep disturbances, Interest loss, Guilt, Energy loss,
Concentration loss, Appetite change, Psychomotor retardation, Suicidal
ideation, LOW MOOD
DYSTHYMIA/Persistent Depressive Disorder (PDD): Low-level depression for greater
than 2 years
In Children/Adolescence: PDD symptoms must be greater than 1 year (not 2)

BIPOLAR I DISORDER: Must have symptoms of mania for GREATER THAN (>) 1 week\
OR if you require hospitalization for manic symptoms
Symptoms: DIG FAST (Distractibility, Insomnia, Grandiosity, Flight of Ideas, Activities,
Speech pressured/more talkative, Thoughtlessness risk-taking behavior)

BIPOLAR II DISORDER: hypomania for at least 4 days


Hypomania: do NOT have social dysfunction (versus Mania: social dysfunction)

Cyclothymia: Depression followed by symptoms of hypomania/mania, depression followed by


symptoms of hypomania/ mania (CYCLES that last for greater than 2 years)
In adolescents/children: Same cycles that last for greater than 1 year
Will NOT have gone for more than 2 months without symptoms

Schizo-symptoms
Brief Psychotic Disorder: Auditory Hallucinations, hearing voices for less than 1 month
Schizophreniform Disorder: Same symptoms as above for 1-6 months (less than 6
months, greater than 1 month)
Schizophrenia: Same symptoms as above for greater than 6 months

Conduct Disorder: MUST BE LESS THAN 18 (criminal activity)


Antisocial personality disorder BUT under 18
NOT THE SAME AS OPPOSITIONAL-DEFIANT DISORDER
Antisocial personality disorder: Same symptoms as conduct disorder BUT over 18

ADHD (Attention Deficit Hyperactivity Disorder): TYPICALLY diagnosed in children under


age 12
Symptoms must be present in at least 2 settings (school, home, etc..)

Panic Disorder: Recurrent panic attacks with worrying about having another panic attack for
OVER 1 month
-------------------------------------------------------------------------------------------------------------------------------
Episode 455 HY Autosomal Dominant Disorders
40 year old male presents to his PCP with dementia, weird arm movements and lower
extremities. The patient says inappropriate things during the history. Dx? Huntington’s Disease
Chromosome 4 Issue; Huntingtin Gene
Trinucleotide Repeat CAG
Chorea commonly (choreiform movements)
Atrophy of Caudate nucleus
ANTICIPATION: future generations will have more severe disease + earlier in life
Number of trinucleotide repeats INCREASES (mechanism behind anticipation)
Pathophysiology: Hypermethylation of Histone Proteins

MOST trinucleotide repeat disorders have AUTOSOMAL DOMINANT inheritance patterns


EXCEPTION: FRIEDREICH'S ATAXIA
Exception: FRAGILE X SYNDROME (Technically X-linked dominant)

Person from Europe comes into the clinic with Jaundice. The patient has an increased
hematocrit with an indirect hyperbilirubinemia. A Coombs test is performed and is negative. Dx?
Hereditary Spherocytosis
Hemolysis NOT due to autoantibodies
Defect in spectrin/ankyrin/band proteins —> insufficient membrane —> super
concentrated hemoglobin
Diagnosis: Osmotic Fragility Test OR Eosin-5’-Maleimide test
Treatment: SPLENECTOMY (splenic macrophages destroys RBCs)

Patient comes in with recurrent nose and gum bleeding. On physical exam, the tongue is
covered in red spots. The patient has elevated cardiac output with elevated EPO. Dx?
Hereditary Hemorrhagic Telangiectasias (Osler-Weber-Rendu Syndrome)
Mucosal Telangiectasias, Epistaxis, AV-malformations, GI bleeds
Elevations in EPO?
Lots of AVMs –> improper gas exchange –> hypoxic tissue –> increases Cardiac
Output (eventual High Output Heart Failure) + increases EPO production
Athlete that dies during a basketball game. Dx? HOCM (Hypertrophic Obstructive
CardioMyopathy)
Mutation in beta-myosin heavy chain –> Sudden Cardiac Death
Young person that collapses during a sports activity
Murmur heard best at the LLSB (Left Lower Sternal Border)
Murmur gets softer when you put more blood in the LV –> pushes the hypertrophic
muscular interventricular septum –> relieves tension in Left Outflow tract
Tx: beta blockers
MOA: Slow down heart –> increases filling of Left Ventricle –> allows for Left ventricular
outflow tract to be opened

Person who has blue sclera. Dx? Osteogenesis Imperfecta (OI)


Issue with Type I collagen
Fractures all over the body

Person that has short limbs compared to their body, abnormal ossification of bones.
Achondroplasia
FGFR-3 (Fibroblast Growth Factor Receptor-3)
Chromosome 4 Issue

2 year old child that is having frequent seizures throughout the day. The child has
hypopigmented skin lesions. Recently the child had a cardiac mass removed. Dx? Tuberous
Sclerosis
Issues with TSC-1 and TSC-2
TSC-1: codes for hamartin
TSC-2: codes for tuberin
Chromosome 16 defects
Hypopigmented skin lesions (ash-leaf spots)
Seizure disorder: Infantile spasms (West Syndrome)
Hypsarrhythmia on EEG
Treatment: ACTH (possibly steroids)
Cardiac masses: Cardiac Rhabdomyomas
Kidney masses: Renal angiomyolipomas
SEGA (subependymal Giant cell Astrocytomas): Developmental delay
“Tubers” in the brain
Adenoma Sebaceum (facial angiofibromas)
Shagreen Patches
Shagreen Patches Ash-Leaf Spots

Hyperpigmented Cafe-au-Lait spots –> Neurofibromatosis

20 year old that gets an MI. Their parent died of a MI at age 32. Dx? Familial
Hypercholesterolemia
Issues with LDL receptor –> cannot clear LDL from circulation –> elevated LDL levels –>
increased risk of MI
Treatment: Statins, PSK-9 inhibitors
Many variations
Familial hyperchylomicronemia

Person that has tons of polyps in the colon. Dx? FAP (Familial Adenomatous Polyposis)
APC gene mutation
Lots of polyps in colon early in life
Turcot syndrome: FAP with brain tumors
Gardner syndrome: FAP with soft tissue tumors

Person has lots of hyperpigmented macules on the LIPS with polyps in the colon. Person may
have a history of pancreatic cancer, hamartomas in the GI tract. Dx? Peutz-Jaghr Syndrome

6 foot 5 guy comes in with severe chest pain. The patient describes the pain as tearing and
feels it in his back. BP is extremely elevated. The patient’s father passed away due to a stroke.
Dx? Marfan’s Syndrome
Chromosome 15 gene disorder
Fibrillin Gene defect
Fibrillin key for extracellular matrix
Dislocated lenses dislocated upwards AND outwards
Mitral valve prolapse
Myxomatous degeneration of the mitral valve
Aortic Dissection (dissect through intima into the media)
Cystic medial necrosis/degeneration
Extremely tall
Pneumothorax (chest pain with no widened mediastinum on Chest X-ray)
Pop apical blebs –> pneumothorax
Restrictive Lung Disease
Pectus excavatum –> lungs do not have enough room to expand –> restrictive
lung disease
Normal DLCO (normal parenchyma), decreased TLC (total lung capacity),
A-a Gradient NORMAL (no issue with the lung)
Why did the father die of a stroke?
Aneurysms in the circle of willis –> aneurysm bursts –> intracerebral hemorrhage

Homocystinuria: Lens dislocated downwards AND inwards


Autosomal Recessive inheritance pattern

Person has a lot of arrhythmias in their family. The patient has frontal balding, and a family
history of cataracts. The testicles are atrophied. Dx? Myotonic Dystrophy
Trinucleotide Repeat Disorder: CTG repeats
DMPK (myotonic dystrophy protein kinase) gene issue
Decreased muscle tone; difficult to release grip (during a handshake)
Frontal balding
Cataracts
Lots of arrhythmias (muscle of heart affected)

Person that has a triad of abdominal pain, neuropsych symptoms, and discolored urine. Dx?
Acute Intermittent Porphyria (AIP)
Issue with porphobilinogen deaminase (deficiency) –> buildup of porphobilinogen in the
blood
Heme synthesis pathway disorder
Port wine urine
Treatment: ALAS inhibitor (heme, hematin, glucose)

Person presents with posterior mediastinal mases, hyperpigmented macules on the skin, and on
CT scan of the brain there are tumors growing near the cerebral convexities. Dx?
Neurofibromatosis Type I (von Recklinghausen’s disease)
Chromosome 17 disorder
Scoliosis
Posterior mediastinal masses: neurofibromas
Hyperpigmented macules on the skin: Cafe-Au-Lait spots
Lisch Nodules in the eye (Hamartomas of the iris)
CN II gliomas (optic gliomas)
Gliomas derived of diencephalic origin
Pheochromocytomas possible (high BP, episodic headache)
Meningioma: mass that grows around cerebral convexities (falx cerebri)
Psammoma bodies (lamellated calcifications) on histology

Lisch Nodules Cafe-Au-Lait Spots

Patient presents with hypertension and hematuria. Both kidneys can be palpated on physical
exam. A murmur is heard at the apex of the heart. Dx? Autosomal Dominant Polycystic
Kidney Disease (ADPKD)
Mutation in PKD-1 gene (Polycystin 1, Transient Receptor Potential Channel Interacting)
Chromosome 16 mutation (more common)
Mutation in PCKD-2 gene (Polycystin 2, Transient Receptor Potential Cation Channel)
Chromosome 4 mutation (15% of cases)
Cysts in the Liver, Pancreas, Kidneys
Can produce EPO –> polycythemia vera
Some cysts can turn cancerous –> renal cell carcinoma
Mitral Valve Prolapse
Aneurysms in the circle of willis
Diverticula of the colon

Person that has extremely dense bones on imaging and is coming to the ED frequently with
bone fractures. Dx? Osteopetrosis (aka Brittle Bone Disease)
Osteoclasts dysfunction due to a mutation in Carbonic Anhydrase Type II
Osteoclast function
Resorption of bone is due to pumping excess acid (H+) into the bone –> leech the bone
–> destroy it
Carbonic anhydrase type II necessary to convert CO2 and water into Carbonic Acid and
Hydrogen
Bones very dense/thick

Patient who has had uterine rupture, and in the past has had pelvic organ prolapse. In the past
the patient has had a history of aortic dissection. Dx? Ehlers-Danlos Syndrome
Type III and Type V collagen defect
Rupture of uterus
DO NOT WANT PATIENTS TO GO INTO NATURAL LABOR (scheduled C-sections)

Newborn with a white reflex (leukocoria). Dx? Retinoblastoma


Rb-gene mutation
Chromosome 13 defect
Osteosarcomas possible in the future (if they have retinoblastomas)

Child has a renal mass. The child also has polycythemia. Previously the child had a cerebral
tumor resected a few years ago. Dx? Von-Hippel-Lindau Syndrome (VHL)
Chromosome 3 problem
VHL: Ubiquitin ligase
If mutated –> cannot ubiquitinated things that should be destroyed (i.e.
HIF-1A; Hypoxia Induced Factor 1 alpha)
Cerebellar hemangioblastomas
Calcified masses in the cerebellum; can secrete EPO –> polycythemia vera
Renal Cell Carcinoma (bilaterally)
Pancreatic Tumors
Pheochromocytomas

Person who bleeds a lot. The PTT is up, bleeding time is elevated, and abnormal ristocetin
cofactor assay. Dx? Von-Willebrand Disease
Deficiency of von-Willebrand Factor (vWF)
Primary platelet problem –> bleeding time increased
vWF binds with GP1b –> primary hemostasis –> abnormal ristocetin cofactor assay
vWF extends efficacy of Factor VIII → PTT elevation (due to ineffective Factor VIII)
Treatment: Desmopressin/ADH-analog (increases release of vWF from the Weibel-Palade
bodies)

Patient who gets lots of cancers (leukemias, adrenal cancers, breast, brain). The patient has a
family history of many cancers. Dx? Li-Fraumeni Syndrome
P53 mutation
Grab-bag of Autosomal Dominant disorders
Lynch Syndrome (Hereditary Nonpolyposis colorectal cancer (HNPCC)
BRCA mutations
Gilbert’s Disease
Antithrombin III Deficiency
Idiopathic Hypoparathyroidism
Protein C Deficiency
Treacher Collins Syndrome
-------------------------------------------------------------------------------------------------------------------------------

Episode 460 Floridly HY Antibiotic Review I


Penicillins (Penicillin G and Penicillin V)
MOA: Inhibit Cell Wall (bind beta-Al tram ring to DD-transpeptidase) —> inhibit cross-linking
activity & prevent new cell wall formation

Michealis-Menten: Do NOT interfere with Vmax, will impact Km (raise Km)


Impact the AFFINITY of substrate for enzyme (decrease affinity of trans-peptidase for its
substrate)

Penicillin uses (most common uses):


Syphilis (treponema pallidum) (IV penicillin)
Rheumatic Fever (Penicillin + NSAID)
Sickle Cell Disease Patient (up till age 5 should be on a Penicillin prophylaxis)
Helps prevent infection by encapsulated organisms (no functional spleen)
S. Pneumoniae (most common cause of sepsis in Sickle Cell patients)

Ampicillin and Amoxicillin (aminopenicillins; broad-spectrum penicillins)


MOA: Inhibit Cell Wall (bind beta-Al tram ring to DD-transpeptidase) —> inhibit cross-linking
activity & prevent new cell wall formation

Ampicillin and Amoxicillin uses:

Amoxicillin Ampicillin

Acute Otitis Media (1st line treatment)’ Listeria Monocytogenes treatment


If it does NOT work —> strongly
consider adding clavulanic acid Meningitis in a neonate & patient over 50
(beta-lactamase inhibitor) (Ceftriaxone + Vancomycin + Ampicillin)
Amoxicillin + Claculonic acid = combo
Augmentin Ceftriaxone to cover gram negative
organisms (N. Meningitidis)
Prophylaxis before dental procedures Vancomycin to cover gram positive organisms
(S. Aureus)
Prophylaxis after dog/cat bite (to prevent Ampicillin to cover listeria monocytogenes
osteomyelitis)

Group B strep prophylaxis in pregnant Chorioamnionitis (CAG treatment)


woman weeks 35-37 Chorioamnionitis: Ampicillin + Gentamicin

Good for UTI treatment (UTIs: cystitis, NOT Prophylaxis before dental procedures
pyelonephritis)
Prophylaxis after dog/cat bite (to prevent
Part of Triple therapy for Helicobacter Pylori: osteomyelitis)
CAP (Clarithromycin + Amoxicillin + Proton
Pump Inhibitor) Group B strep prophylaxis in pregnant
woman weeks 35-37
Adverse effect of Amoxicillin: In a mono
patient —> full body rash Good for UTI treatment (UTIs: cystitis, NOT
pyelonephritis)

Endometritis (Infection of uterus; 3 days after c-section delivery): (ECG treatment)


Endometritis: Clindamycin + Gentamicin

Prophylaxis with Amoxicillin/Ampicillin before dental procedures IMPORTANT for patients with
heart defect or previous history of endocarditis or mechanical valve in the heart
KEY indication
SINGLE dose

Mainstay treatment for UTIs in women: Nitrofurantoin/TMP-SMX


(trimethoprim-sulfamethoxazole)
Ampicillin/Amoxicillin 2nd line therapy

Penicillinase-resistant Penicillins (Methicillin, Dicloxacillin, Oxacillin, Nafcillin)


Cover MSSA (methicillin-sensitive staph aureus)
RARELY used in real world
Methicillin NOT used
Uses of Dicloxacillin/Nafcillin/Oxacillin:
Mastitis (usually caused by MSSA)
Nafcillin Adverse Effect: Strong association with Acute Interstitial Nephritis (AIN)
AIN: Elevated Eosinophils + Rash + Fever
Antipseudomonal penicillins (Piperacillin + Tazobactam, Aztreonam, Carbapenems,
Fluroquinolones, Aminoglycosides, 3rd gen Cephalosporins, 4th gen cephalosporins)
Pipercillin with Tazobactam: Combination (Tazobactam = beta-lactamase inhibitor)
Aztreonam: Monobactam (rarely used on exams); good in people with penicillin allergies
Carbapenems: Imipenem, Meropenem, Doripenem
Fluoroquinolones: Ciprofloxacin, Levofloxacin
3rd gen cephalosporins: Ceftazidime, (ceftriaxone/cefotaxime do NOT cover P. Aeruginosa)
4th generation cephalosporin: Cefepime

Cephalosporins
1st generation: Cefazolin, Cephalexin
Cefazolin: Antibiotic of choice PRIOR to surgery (30-60 min prior)
Cephalexin: Useful in skin infections (cellulitis, erysipelas), UTIs (SAFE IN
PREGNANCY)
2nd generation: Cefuroxime
Cefuroxime: Good for treatment of Lyme disease (NOT 1st line; Doxycycline 1st line)
3rd generation: Ceftriaxone, Cefdinir, Cefotaxime, Cefixime, Ceftazidime, Ceftizoxime
Ceftriaxone: DOC for pyelonephritis, DOC for N. Gonorrhea infection, N. Meningitidis,
MOST GI infections (MCC of GI infections: E. Coli), Spontaneous Bacterial Peritonitis (SBP;
ascites, mild fever, mild abdominal pain, altered mental status), Close contact prophylaxis for
people exposed to N. Meningitidis (spreads by respiratory droplets), Pneumonia (S. Pneumo, H.
Influenza)
CHELATES calcium —> biliary stasis (biliary issues)
Cefotaxime: ANYTHING you can use for ceftriaxone; Preferred in small children over
ceftriaxone; sepsis in a neonate; SBP
Ceftazidime: Covers PSEUDOMONAS AERUGINOSA
4th generation: Cefepime
Covers Pseudomonas
Adverse Effect: Abdominal pain, Diarrhea
5th generation: Ceftaroline
Covers MRSA (Methicillin-Resistant Staph Aureus)

Lyme Disease: Caused by Borrelia Burgdorferi


Bacteria carried by the Ixodes Tick (also Carries Babesia Microti, Anaplasma)

DOC for neutropenic Fever: ANTIPSEUDOMONAL ANTIBIOTIC

Close contact prophylaxis for N. Meningitidis: Ceftriaxone (preferred in pregnancy),


Ciprofloxacin, Rifampin (Preferred agent overall)

Best respiratory antibiotics: Fluoroquinolones (levofloxacin/moxifloxacin), Ceftriaxone


Vancomycin
MOA: Inhibits cell wall synthesis by binding to D-Ala-D-Ala terminal growing peptide chain
during cell wall synthesis —> inhibit transpeptidase —> Prevent further elongation &
crosslinking of peptidoglycan matrix
Uses of Vancomycin:
MRSA (Methicillin-Resistant Staph Aureus)
Empiric treatment of Meningitis (if bug unknown; WITH ceftriaxone)
C. Difficile treatment (oral vancomycin)
Staph Epidermidis
Adverse Effects: REDMAN syndrome

Fidaxomicin: useful for treatment of C. Difficile

Carbapenems: Doripenem, Imipenem, Ertapenem


Given to a patient with SERIOUS infections (ICU/Critical Care level)
Most gram negatives, Most gram positives
NO MRSA COVERAGE
Aztreonam: Pseudomonas coverage

Polymyxin B and Polymyxin E (colistin)

Mupirocin: Useful for impetigo & folliculitis


Useful for MRSA when in the nose
-------------------------------------------------------------------------------------------------------------------------------

Episode 461 Floridly HY Antibiotic Review II


Aminoglycosides: Gentamicin, Tobramycin, Amikacin, Streptomycin, Neomycin,
Paromomycin
MOA: Bind 30S subunit of bacterial ribosome –> inhibit protein synthesis of cell wall –>
cell death (bactericidal)
Uses: Pseudomonas, Tularemia, Meniere’s Disease (will destroy CN VIII)
Gentamicin: part of treatment regimen for endometritis (fever + uterine
tenderness a few days AFTER c-section); part of treatment for Pseudomonas
Aeruginosa; treatment of endocarditis
Inhaled Tobramycin: Given to CF patients as prophylaxis for pseudomonas
CANNOT be used for ANAEROBIC INFECTIONS
Adverse Effects: Nephrotoxicity, Ototoxicity
Neomycin can cause a hypersensitivity reaction (TYPE IV)
Ability to block neuromuscular receptors (nicotinic acetylcholine receptors) –> muscle
weakness (neuromuscular blocking agents)
Myopathy AFTER gentamicin (especially in ALS/MELAS/Myasthenia Gravis)

ECG for Endometritis: Endometritis is treated with Clindamycin and Gentamicin

Most endocarditis treatment regimens: Vancomycin, Gentamicin, and Rifampin

Exposure to rabbits –> Francisella Tularensis

Lyme Disease Drugs (aka Tetracyclines): Doxycycline, Minocycline, Tetracycline,


Tigecycline, Demeclocycline
MOA: Inhibit 30S subunit (hinder binding of aminoacyl t-RNA to acceptor site on
mRNA-ribosomal complex) –> interfere with cell protein synthesis –> inhibit cell
reproduction (bacteriostatic)
DOC for chlamydia infections, can cover MRSA
DOC for Rocky Mountain Spotted Fever (RMSF;Rash on palm and soles, R. Ricketsii)
DOC for H. Pylori Infection
Triple therapy: Clarithromycin, Amoxicillin, PPI
Quadruple therapy: Bismuth, Metronidazole, Tetracycline, PPI
Treatment for Lyme Disease (everyone gets a tetracycline; independent of age)
Somewhat contraindicated in pregnancy (can give cefuroxime or amoxicillin)
Treatment of Acne Vulgaris
Can be chelated by antacids
Adverse Effects: Photosensitivity, Trigger for Idiopathic Intracranial Hypertension (i.e.
Pseudotumor Cerebri), teratogens, issues with bones in children, teeth discoloration
Can be seen in obese female patient being treated for acne who presents with
morning headaches, blurry vision for the past few weeks

Macrolides can be used for chlamydia (however less effective and not first line; more treatment
failures)

Drugs that can be chelated by Antacids


Fluoroquinolones
Levothyroxine
Bisphosphonates

Drugs that can cause photosensitivity (SAT for photo)


Sulfonamides
Amiodarone
Tetracyclines

Macrolides: Erythromycin, Clarithromycin, Azithromycin, Fidoxamicin, Roxithromycin,


Telithromycin
MOA: Bind to the 50S ribosomal subunit of bacteria –> inhibit protein synthesis → stop
bacterial reproduction (bacteriostatic)
Uses
Part of Triple therapy for H. Pylori: Clarithromycin, Amoxicillin, PPI
2nd line agent for Chlamydia infection
Walking Pneumonia (interstitial pneumonia in persons age 20-40, nonproductive
cough, fatigue, malaise, low-grade fever)
Usually caused by Mycoplasma Pneumoniae
Azithromycin good as 1st line treatment
COPD exacerbation + interstitial infiltration
Azithromycin good for treatment of atypical organisms + has anti
inflammatory properties
Pertussis (Cough + [Blood (subconjunctival hemorrhage)/vomit/syncopal episode
after coughing]
Diabetic Neuropathy of the stomach (gastroparesis)
Erythromycin is a motilin receptor AGONIST (encourage motility of GI
tract)
Legionella Pneumophila (Legionaire’s disease (smokers with interstitial
pneumonia; pontiac fever; May occur with water exposure (humidifiers, HVAC,
waterfalls) OR can be an outbreak in a hospital)
Prophylaxis for Neonatal Conjunctivitis (Topical erythromycin/tetracycline)
GONOCOCCAL conjunctivitis, NOT chlamydial conjunctivitis
Adverse Effect: QT interval prolongation

Treatment options for diabetic gastroparesis


Metoclopramide (adverse effects: extrapyramidal effects)
Erythromycin

Chloramphenicol
Adverse Effect: Gray-Baby Syndrome
Rarely used on exams

Clindamycin
MOA: Protein Synthesis inhibitor
Uses:
Anaerobic infections
Necrotizing Fasciitis (due to the toxin in the bacteria causing the illness)
Endometritis (ECG: Endometritis treated with Clindamycin + Gentamycin)
MRSA coverage
Side Effects: C. Difficile Colitis (due to destroyed GI tract),

Linezolid
MOA: Binds 50S ribosomal subunit –> prevents formation of 60S ribosomal complex
(50S/30S complex)
Uses
MRSA and VRSA
Adverse Effect: MAOI effects (can trigger serotonin syndrome IF taken with another
serotonergic drug)

Fluoroquinolones: 3 generations
1st generation: Norfloxacin, Nalidixic Acid, Cinoxacin
2nd generation: Ciprofloxacin, Ofloxacin, Lomefloxacin
3rd generation: Levofloxacin, Gatifloxacin, Moxifloxacin
4th generation: Trovafloxacin
Attempt to avoid using them (if possible)
Adverse effects: QT prolongation, Tendon rupture
MOA: Inhibit DNA gyrase
Cover Gram negatives well (i.e. E. Coli)
Uses:
Cover Pseudomonas (good coverage)
Many respiratory infections (levofloxacin/moxifloxacin)
SBP prophylaxis (in patients with variceal bleeds, previous SBP)
SBP: Ascites, abdominal pain, fever, slight/subtle altered mental status

Ethacrynic Acid (loop diuretic)


Can be used in patients with SULFA allergies
All other loop diuretics: Furosemide, Torsemide, Bumetanide
Barbiturates: Primidone (can be used to treat Restless leg syndrome)
All other barbiturates: Phenobarbital, Thiopental, Secobarbital, Pentobarbital, Amobarbital

Benzodiazepine: Chlordiazepoxide (helps prevent alcohol withdrawal)


All other benzodiazepines: Oxazepam, Lorazepam, Diazepam, Alprazolam, Clorazepate,
Prazepam

Metronidazole
MOA: Binds DNA to block nucleic acid synthesis
Covers Anaerobes very well
2nd/3rd line for agent for C. Difficile (Vancomycin 1st line)
Uses
Part of Quadruple therapy of H. Pylori: PPI, Tetracycline, Metronidazole, Bismuth
OBGYN infections
bacterial vaginosis (vaginal pH greater than 4.5, “clue cells” on cytology,
“fishy smell)
Trichomoniasis (motile protozoa on cytology; vaginal pH greater than 4.5)
“GET GAHP on the metro”
Gardnirella Vaginalis
Entomeaba Histolytica (blood diarrhea, liver abscess)
Trichomoniasis
Giardial Lamblia (Long-term traveler’s diarrhea; common in IgA
deficiency/Bruton’s Agammaglobulinemia/CVID)
Anaerobes (Clostridium, Bacteroides, actinomyces)
H. Pylori
Adverse Effect: Disulfiram-Reaction (can inhibit acetaldehyde dehydrogenase)
DO NOT DRINK ALCOHOL WITH METRONIDAZOLE

Tuberculosis Drugs: Rifampin, Isoniazid, Pyrazinamide, Ethambutol (RIPE therapy)


Active TB: Symptomatic; RIPE regimen + Vitamin B6
Isoniazid can cause a Vitamin B6 deficiency
Rifampin used as prophylaxis for close contacts of N. Meningitidis infected people
Rifampin OCCASIONALLY used for Endocarditis (part of the regimen)
Rifampin can be used to treat Mycobacterium Leprae (for 2 years)
Latent TB: (positive TB skin test/quantiferon assay + Normal chest X-ray) Isoniazid +
vitamin B6
Rifampin used as prophylaxis for close contacts of H. Influenzae infected patients
Ethambutol used as treatment of Mycobacterium Marinum Infection, Mycobacterium
Avium Complex Infection
Adverse Effects:
Rifampin-Neurotoxic and Hepatotoxic; Turns secretions orange/red;
Isoniazid-Neurotoxic and Hepatotoxic; Peripheral Neuropathy; Vitamin B6
deficiency
Pyrazinamide- No significant adverse effect
Ethambutol- Red-Green colorblindness (optic neuropathy)

Sulfonamides: Trimethoprim-Sulfamethoxazole
Uses:
Pneumocistis Jirovecii (treatment AND prophylaxis IF CD4 < 200)
MRSA skin infections
Toxoplasmosis (T. Gondia infection) Prophylaxis IF CD4 < 100
Toxoplasmosis infection treatment: Pyrimethamine-Sulfadiazine
Many UTIs (Cystitis, Pyelonephritis)
Cover Gram (-) bacteria well
MOA: Trimethoprim and sulfamethoxazole inhibit bacterial synthesis of tetrahydrofolic
acid (Trimethoprim (TMP) prevents reduction of dihydrofolate to tetrahydrofolate.
Sulfamethoxazole (SMX) inhibits conversion of p-aminobenzoic acid to dihydropteroate)
Adverse Effect: Hyperkalemia; Drug-Induced Lupus
Contraindicated in People with G6PD deficiency (trigger a hemolytic crisis)

Leprosy Drug Regimen: Rifampin + Dapsone + Clofazimine for 2 years

Mycobacterium Avium Complex/Mycobacterium Marinum Triple Drug Regimen (CER)


Clarithromycin + Ethambutol + Rifampin

Digoxin Adverse Effect: Optic Neuropathy (halos around objects; green vision)

Hydroxychloroquine: Retinopathy
Requires yearly eye exams

Drug Induced Lupus triggers


Isoniazid
Hydralazine
Phenytoin
Etanercept
Sulfonamides
Procainamide
Quinidine

Voriconazole: Antifungal
Treats MOST fungal infections
Effective against invasive aspergillosis

Amphotericin B: Antifungal
Treat Mucormycosis
2nd line treatment for invasive candidal infections
2nd line treatment of invasive aspergillosis
Treatment of HIV patient associated Cryptococcal meningitis
Given WIth 5-Flucytosine; Flucanozole after 12 months (to prevent recurrence)
Liposomal Amphotericin B
Used to treat cutaneous Leishmaniasis

Paromomycin & Nitazoxanide:


Used to treat diarrhea in patients with Cryptosporidium infections

Itraconazole: Antifungal
Used to treat dermatophyte infections (tinea cruris/tinea pedis)
Used to treat endemic fungal respiratory infections
Histoplasmosis, Blastomycosis, Coccidiomycosis

Topical Permethrin
Used to treat SCABIES

Oral terbinafine/Oral Griseofulvin: Can penetrate keratinocytes


Treat Tinea Capitis/Onychomycosis
Griseofulvin: Cytochrome p450 inducer
Increases rate of metabolism of other drugs

CYP450 inducers
Griseofulvin
Barbiturates
Rifampin
St John’s Wort
Phenytoin
Valproic Acid
-------------------------------------------------------------------------------------------------------------------------------

You might also like